You are on page 1of 56

1

II Bienal da SBM
Salvador/BA - 25 a 29 de Outubro de 2004

Olimp ada de Matemtica - Uma porta para o futuro a

Dicas para montar um projeto e 50 problemas de treinamento para iniciantes

Emanuel Carneiro

Apresentao ca
Os livros nos contam que, em tempos mais antigos, matemticos desaavam uns aos outros propondo questes complia o cadas e por muitas vezes se reuniam em praa pblica para a realizaao de torneios, onde teriam que resolver equaoes c u c c dif ceis. O que nasceu talvez por um capricho do ego destas pessoas tomou forma mais salutar com a realizaao da 1a c Olimp ada de Matemtica, na Hungria em 1896. a De l pra c, as competioes de matemtica entre estudantes vm a cada dia se organizando e se mostrando um a a c a e forte indicador para descobrir novos talentos para a cincia. Em 1959, foi realizada a 1a Olimp e ada Internacional de Matemtica (IMO), com bero no leste europeu e contando apenas com a presena de pa comunistas. Em 1965 a a c c ses Finlndia se torna o primeiro pa no comunista a fazer parte do hall de participantes da IMO, abrindo o caminho a s a para que esta competiao viesse a ser a maior de matemtica do mundo atual (hoje na sua ediao 46), com cerca de c a c 85 pa participantes por ano e que rene em mdia 500 estudantes de todo o planeta. ses u e Esse evento mundial estimulou a criaao de plos em cada regio. Assim, hoje em dia, cada pa conta com sua c o a s Olimp ada Nacional de Matemtica, e por um racioc a nio anlogo, as prov a ncias destes pa tambm resolveram orses e ganizar competioes de matemtica e at os prprios munic c a e o pios. Foi assim que tudo comeou em nosso pa Em 1979 foi realizada a 1a Olimp c s. ada Brasileira de Matemtica, por a iniciativa amadora de alguns matemticos. A OBM est hoje em sua 26a ediao, abrangendo cada vez lugares mais a a c remotos em nosso pa contando com a participaao anual de cerca de 150.000 alunos. Vrios estados brasileiros s, c a tambm j contam hoje com sua prpria Olimp e a o ada de Matemtica, numa prova de evidente crescimento do interesse a das pessoas por estas competioes. c Por muito tempo, o trabalho de se organizar as competioes de matemtica em nosso pa foi completamente amador, c a s feito por admiradores incondicionais da matemtica e de seus segredos. Apenas h alguns anos (em 1998), uma verba a a do governo foi destinada para ns de apoio, e com isso a OBM cresceu em estrutura e em nmero de adeptos. O u resultado do Brasil na IMO vem se consolidando a cada ano, hoje ocupando posiao de destaque entre os 20 melhores c pa do mundo. Sim, ao contrrio do que muitas pessoas pensam, o Brasil bom em matemtica, sim! Por enquanto ses a e a talvez s na matemtica de ponta, mas o mais importante que isso foi conquistado em relativamente pouco tempo, o a e o que nos impulsiona a acreditar mais e mais no POTENCIAL que o brasileiro tem para aprender. Este texto sobre isso. Sobre o POTENCIAL de nossos estudantes que cam em sua maioria encurralados por e um ensino pblico decadente. Sobre a entidade Olimp u ada de Matemtica que cresce em nosso pa como instrumento a s, de desenvolvimento da educaao e processo de incluso social indiscut pois oferece muitas oportunidades a jovens c a vel independentemente de raa, sexo e lugar onde mora. c Gostaria de apresentar a vocs minha viso das coisas e mostrar o que se pode fazer com um pouquinho de dedicae a cao e vontade de melhorar. Quero apresentar a Olimp ada de Matemtica no como um m, mas como um dos meios, a a uma opao, para reerguermos o ensino de matemtica no Brasil. Da o t c a tulo do texto.

Emanuel Carneiro - Fortaleza - Setembro de 2004

Cap tulo 1

O que a matemtica de Olimp a ada tem de diferente: alguns exemplos


At aqui, j falamos um pouco sobre a Olimp e a ada de Matemtica, o histrico e as competioes existentes no Brasil e no a o c mundo. Chegou ento a hora de conhecer um pouco dos problemas que costumam compor as provas destas competioes. a c Erroneamente, muitas pessoas pensam que estudar para participar de uma Olimp ada de Matemtica avanar na a e c matria usual do colgio, que por exemplo um aluno da 5a srie deva estudar equaao do 2o grau, ou um aluno do e e e c ensino mdio deva saber tudo de clculo. No nada disso. Os problemas no exigem uma dose maior de conhecie a a e a mento, e sim o despertar de um racioc nio e de muita criatividade. O aluno forado a experimentar sua inteligncia, e c e diferentemente da maioria dos problemas mecnicos que fazem parte do curr a culo escolar. Como primeiro exemplo, o seguinte problema foi proposto na Olimp ada de Matemtica da Escola Pblica do Esa u tado do Cear em 2003, na modalidade II (1o ano do ensino mdio): a e Exemplo 1.0.1 So dadas 4 moedas aparentemente iguais. Sabe-se que uma delas falsa (tem peso diferente das a e demais). Mostre como descobrir a moeda falsa com 2 pesagens em uma balana de dois pratos. c Tentem pensar alguns minutos antes de olhar a soluao... c Soluo: Chame as moedas de A, B, C, D. Pese duas delas, digamos A e B. Temos ento duas possibilidades: ca a H equil a brio entre A e B: Nesse caso A e B devem ser moedas leg timas. Pese ento A com C. Se equilibrar a de novo, a falsa dever ser a moeda D. Se no equilibrar, C ser a falsa. a a a No h equil a a brio entre A e B: Conclu mos da que A ou B a falsa. Pese ento A e C (que uma moeda e a e verdadeira). Se houver equil brio, B ser a falsa. Se desequilibrar, A ser a falsa. a a De qualquer modo voc consegue ento descobrir qual a moeda falsa usando duas pesagens. e a Comentrios: Aps ter visto a soluao deste problema, podemos destacar alguns pontos: a o c Apesar de ter sido proposto para a modalidade II (1o ano do ensino mdio), este problema poderia ser resolvido e por alunos do ensino fundamental, j que so necessrios apenas racioc a a a nio e um pouco de pacincia. e A maioria dos alunos ao olhar um problema deste tipo pensa 1 minuto e diz que no sabe fazer. Isso ocorre a porque eles esto acostumados com uma matemtica rpida, de muita conta e pouca idia. Se o aluno for a a a e devidamente estimulado (oferea um prmio!) certamente ele vai fazer. c e Como foi visto na soluao, o aluno ao fazer uma prova de Olimp c ada de Matemtica, deve tambm ter um bom a e dom nio do portugus, j que o unico modo que ele tem para expor suas idias atravs do lpis e do papel. e a e e e a Muitas vezes alunos saem das provas dizendo: - Ah professor, essa questo eu sabia fazer, s no consegui explicar direito... a o a

No adiantou nada. A pessoa que vai corrigir vai pensar que ele no sabia. Resumindo, est em jogo aqui a a a no s a capacidade de racioc a o nio do aluno, mas tambm sua capacidade de comunicaao escrita e a sua orgae c nizaao (anal, s ns professores sabemos como chato corrigir provas cheias de borres inndveis...) c o o e o a Aps este primeiro contato, podemos ento ver um segundo exemplo. o a Exemplo 1.0.2 (i) Mostre como dispor os nmeros 1, 2, 3, 4, 5, 6, 7, 8, 9 nas casas de um quadrado 33 para formar u um quadrado mgico, ou seja, para que a soma dos nmeros nas linhas, colunas e diagonais seja a mesma. a u (ii) Prove que em todo quadrado mgico o nmero da casa central deve ser o 5. a u Novamente, pense um pouco antes de ver a soluao. c Soluo: (i) H vrias maneiras. Por exemplo: ca a a 4 9 2 3 5 7 8 1 6 6 7 2 1 5 9 8 3 4

(ii) Digamos agora que a soma comum (das linhas, colunas e diagonais) seja S. Vejamos ento uma distribuiao a c qualquer para um quadrado mgico: a a d g Somando as trs expresses temos: e o a + b + c + d + e + f + g + h + i = 3S 1 + 2 + 3 + 4 + 5 + 6 + 7 + 8 + 9 = 45 = 3S S = 15 Por enquanto, s usamos as somas dos nmeros de cada linha. Temos ainda uma porao de somas que podemos o u c utilizar. Estrategicamente escolhemos as seguintes: a + e + i = 15 g + e + c = 15 d + e + f = 15 b + e + h = 15 Somando estas ultimas quatro expresses, obteremos: o (a + e + i) + (g + e + c) + (d + e + f ) + (b + e + h) = 60 (a + b + c + d + e + +f + g + h + i) + 3e = 60 (1 + 2 + 3 + ... + 9) + 3e = 60 45 + 3e = 60 e=5 b e h c f i a+b+c d+e+f g+h+i = S = S = S

Mgica? a

O que vocs acharam ento deste segundo exemplo? O primeiro item um timo exerc e a e o cio para qualquer idade. Imagina a felicidade de um aluno de 10 ou 11 anos que consiga resolv-lo...J o segundo item um pouco mais elaboe a e rado. O aluno pode ento perguntar: a -Professor, me diga como que eu conseguiria fazer isso? Eu jamais teria essas idias. e e

- Tudo bem, voc no sabia. Agora eu j mostrei pra voc. Da prxima vez ento que aparecer algo assim, voc e a a e o a e j pode comear a ter estas idias... a c e A melhor forma de uma pessoa aprender matemtica resolvendo exerc a e cios. Quando no sabemos resolver algo, a isso no deve ser o m do mundo, anal ningum obrigado a saber tudo. Neste ponto devemos chegar para algum a e e e mais experiente para que esta pessoa nos d novas idias, para que ela nos passe o bizu e nos ensine a resolver o e e maldito problema. Deste momento em diante, teremos estas novas idias em nosso arsenal e quando aparecer outro e problema parecido... O prximo exemplo um pouco mais elaborado, mas no menos interessante: o e a Exemplo 1.0.3 Mostre que no h inteiros a a mpares a, b, c, d, e, f tais que: 1 1 1 1 1 1 + + + + + =1 a b c d e f Soluo: Se isto fosse poss ca vel, ter amos: bcdef + acdef + abdef + abcef + abcdf + abcde =1 abcdef bcdef + acdef + abdef + abcef + abcdf + abcde = abcdef

Nesta ultima expresso temos do lado esquerdo a soma de seis nmeros a u mpares, que sabemos que par. Porm, do e e lado direito temos um nmero u mpar (abcdef ). Absurdo!! Comentrios: A soluao acima j um tanto quanto mais sosticada. No que seja dif a c a e a cil, pois acredito que a maioria das pessoas est apta a compreend-la. Porm, a soluao usa duas ferramentas bsicas em matemtica: o a e e c a a mtodo de reduao ao absurdo e os argumentos de paridade (um nmero ou par ou e c u e mpar, jamais os dois ao mesmo tempo). Se o aluno no conhece estas ferramentas, ele vai certamente encontrar diculdade. Novamente voltamos ` cona a versa do exemplo anterior. Cabe ento ao professor ensinar as armas bsicas (e nestas o absurdo e a paridade esto a a a certamente inclu das) para que seus pupilos possam achar estas idias naturais. e Um fato curioso que a maioria dos alunos conhece a paridade dos nmeros (anal eles devem jogar par-ou- e u mpar para sobreviver), mas cam extremamente abismados quando vem que argumentos de paridade possam ser usados e para resolver muitos problemas de matemtica. a No prximo exemplo vemos um tpico que est entre os mais queridos na matemtica, e que aparece com freqncia o o a a ue nas competioes: geometria. c Exemplo 1.0.4 No tringulo ABC abaixo, BP bissetriz do angulo B e M o ponto mdio do lado AC. Se AB = 6 a e e e e BC = 10, calcule PM. B

6 P A M

10

Soluo: Veja a nova gura, onde prolongamos AP at encontrar o lado BC em Z. ca e

6 P A M

10

Note que no tringulo ABZ, o segmento AP altura e bissetriz. Isso faz com que o tringulo ABZ seja issceles! Logo a e a o BZ = AB = 6 e portanto: ZC = BC BZ = 10 6 = 4 Perceba ainda que como o tringulo ABZ issceles, BP altura, bissetriz,... e mediana! Logo P o ponto mdio de a e o e e e AZ. Como M j o ponto mdio de AC, vemos que PM a base mdia no tringulo AZC. Concluso: ae e e e a a PM = ZC =2 2

E este agora, o que acharam? A geometria plana um dos tpicos que mais cativam os estudantes. Acho que eles e o se sentem felizes fazendo desenhos. Isso bom, pois na prtica 30% dos problemas propostos em olimp e a adas de matemtica so de geometria. a a Agora vamos a alguns exemplos voltados para alunos do ensino mdio. Anteriormente vimos problemas de matemtica e a que envolviam muito racioc nio e criatividade, mas nem sempre isso pode ser suciente. Se um aluno deseja participar da Olimp ada de Matemtica e obter sucesso, uma condiao necessria que ele tenha o dom a c a e nio da matria e do colgio, dos fundamentos de matemtica. Que ele tenha certa base. Isso mais notvel quando mostramos os e a e a problemas ol mpicos para o ensino mdio, pois nesses o aluno deve aliar sua inteligncia ` sua maturidade matemtica. e e a a O prximo exemplo muito bacana. Foi questo da Olimp o e a ada Brasileira de Matemtica, 1a fase, em 2002. a Exemplo 1.0.5 Seja a maior raiz da equaao x2 x 1 = 0. Calcule 5 5. c Este realmente vale 5 minutos do seu tempo... Soluo: Usando o fato que 2 1 = 0, temos: ca 2 4 Assim conclu mos que: 5 5 = 3 Comentrios: a Muitas vezes as pessoas esquecem de usar a prpria deniao de raiz de uma equaao. Que este um nmero o c c e u que posto no lugar do x vai zerar a equaao. Foi isso que zemos na soluao. c c A maioria dos estudantes, quando escuta a palavra raiz, vai longo chamando a frmula de Bhskara para o a encontrar: 1+ 5 = 2 e vai fazendo um monto de contas... a
3

+1

= ( + 1) = 2 + = 2 + 1 = (2 + 1) = 22 + = 3 + 2 = (3 + 2) = 32 + 2 = 5 + 3

Isso no impede que ele resolva o problema. A entra o que dissemos anteriormente. Se ele for um bom aluno a do ensino mdio e tiver aprendido binmio de Newton ele vai conseguir calcular e o resposta. Vai dar s um pouquinho mais de trabalho... o
1+ 5 2 5

, e vai encontrar a

Para encerrar ento esta pequena amostra de problemas interessantes em matemtica, a vai um bem legal: a a Exemplo 1.0.6 Prove que a funao f : R R denida por: c

f (x) = cos x + cos(x 2)

no peridica. a e o Soluo: Vamos novamente usar o mtodo de reduao ao absurdo (isso realmente importante!). ca e c e Suponha que nossa funao f seja peridica de per c o odo p. Isso quer dizer que: f (p + x) = f (x) para qualquer x R. Em particular, tomando x = 0, temos:

f (p) = f (0) cos p + cos(p 2) = cos 0 + cos 0 = 2 cos p = cos(p 2) = 1

Da devemos ento obrigatoriamente ter: a

Porm, como somos estudiosos da trigonometria, sabemos que: e cos p = 1 p = 2k1 cos(p 2) = 1 p 2 = 2k2 onde k1 e k2 so inteiros (nenhum dos quais zero, pois p no zero, j que f claramente no-constante) Dividindo a e a e a e a ento estas duas ultimas expresses, obtemos: a o p 2 2k2 k2 2= = = p 2k1 k1 e u Chegamos ento a um absurdo, pois sabemos que 2 um nmero irracional a Espero que vocs tenham gostado destes diversos problemas, e mais importante ainda, que tenham percebido as e semelhanas e diferenas entre os problemas de olimp c c ada e os problemas de sala de aula. Penso tambm que agora vocs estejam convencidos que uma pessoa no precisa ter estudado at o terceiro ano e e a e da faculdade para fazer estes problemas. Eles so acess a veis a todos. Esto encerrados portanto os trabalhos deste cap a tulo.

10

Cap tulo 2

Como montar um projeto Olimp ada de Matemtica na sua escola a


Este cap tulo talvez seja o mais importante de todos. Desejo aqui passar a vocs algumas dicas de como implementar e um projeto de Olimp ada em suas escolas, sejam elas pblicas ou privadas. u Para facilitar o entendimento das pginas que se seguem, resolvi dividir o cap a tulo em vrios tpicos, que listo a a o seguir: 1. O que um projeto de Olimp e ada? 2. Quais as vantagens de se ter um projeto como este na escola? 3. Que professores/alunos devem participar? 4. Como denir as turmas e os horrios das aulas. a 5. Material necessrio. a 6. Sugesto de programas didticos. a a 7. Posso sofrer rejeiao de alunos/professores da escola? c 8. O que eu (professor) ganharei com isso? 9. O que meus alunos ganharo com isso? a 10. Montando uma mini-biblioteca. 11. Contratempos que podem acontecer. 12. Como dar continuidade ao processo? 13. Est mulo constante aos seus alunos. 14. Estreitando os laos com SBM e as universidades. c Vamos ento discutir estes tpicos um a um. O que ser dito aqui no pode ser tomado como certeza de sucesso futuro, a o a a so apenas algumas dicas que vo servir como guia para que os interessados possam iniciar/continuar um trabalho a a semelhante. Vamos ao que interessa!

11

12

2.1

O que um projeto de Olimp e ada?

J discutimos nesse livreto vrios tpicos relacinados com as competioes matemticas: seu histrico, bases pedaggicas, a a o c a o o exemplos de problemas...mas o que vem a ser um projeto de Olimp ada de Matemtica na escola? a Este projeto consiste na preparaao de alunos para competioes de matemtica. c c a Um grupo de professores de matemtica da escola vai se reunir e iniciar a preparaao de algumas turmas para as a c competioes matemticas. Ensinando o qu? Obviamente, matemtica! Porm no uma aula comum, e nem um c a e a e a reforo escolar. Os tpicos a serem abordados so aqueles exemplicados no cap c o a tulo anterior (vamos j discutir mais a sobre isso), de uma maneira mais informal, com o lao professor-aluno mais estreito, de modo que as pessoas que se c renam ali, possam desfrutar do simples prazer de discutir matemtica. u a

2.2

Quais as vantagens de se ter um projeto como este na escola?

O principal objetivo do projeto Olimp ada de Matemtica desenvolver o ensino de matemtica na escola. Acreditaa e a mos que projetos como este so uma porta para o futuro, uma das alternativas de melhorar como um todo a educaao a c em nosso pa s. Como este processo funciona? O ensino de matemtica em sala de aula e nas aulas para olimp a ada devem funcionar de maneira harmoniosa, um complementando o outro, de preferncia at ministrado pelos mesmos professores. O aluno que frequenta as aulas de e e olimp ada, vai ter oportunidade de estar em contato com novas idias da matemtica, e isso certamante vai estimular e a seu racioc nio e sua criatividade. Rapidamente o seu rendimento escolar vai melhorar, no s em matemtica, mas tambm nas outras matrias. Ele vai a o a e e virar ento um referencial na sala, sendo respeitado pelos colegas, vai tirar dvidas, etc... e isso vai fazer com que os a u prprios colegas se sintam estimulados a assistir as aulas de olimp o ada tambm. Com o passar do tempo o professor e vai notar que o embasamento dos alunos melhorou, e ele mesmo vai se sentir mais estimulado a ensinar, a procurar coisas mais interessantes para passar aos alunos. Com isso ele vai tambm estudar e aprender mais. E assim o ciclo e recomea com professores e alunos, ainda mais estimulados. c Observe que o mesmo processo acontecer se forem ofertadas aulas para olimp a ada de qu mica, f sica, biologia, informtica, astronomia... a Por que voc diz que a participaao do aluno na olimp e c ada de matemtica tende a melhorar seu rendimento em a sala de aula, tanto em matemtica como nas outras disciplinas? a As competioes de matemtica so em geral bem mais dif c a a ceis que as provas do colgio. E um princ e pio simples: duas pessoas vo participar de uma corrida de 3km. Um deles treina exatos 3km todo dia, enquanto o outro treina a 5km. No dia da corrida quem vai sentir mais facilidade? Ao treinar para fazer uma prova mais dif o seu rendimento cil, nas provas do colgio vai certamente aumentar. e E nas outras disciplinas? Bem, no porque ns somos matemticos, mas de certa forma todos devem concora e o a dar que matemtica e o portugus desempenham um papel essencial no processo educacional. E se uma pessoa tem a a e oportunidade de desenvolver o racioc nio e o lado criativo, com certeza isso o ajudar diretamente em outras cincias a e exatas com f sica e qu mica, por exemplo. Os projetos de Olimp ada de Matemtica nas escolas so relativamente novos, tm no mximo 10 anos, tanto em a a e a minha cidade (Fortaleza) como em So Paulo, que foram as pioneiras. O que houve na cidade de Fortaleza foi um a grande salto da educaao local, que nos ultimos anos vem conseguindo resultados expressivos a n nacional. c vel Obviamente, no tenho a pretenso de apontar a Olimp a a ada de Matemtica como unico responsvel por isso, mas a a devo inclu -la, juntamente com os demais projetos de olimp ada cient cas, como fator importante para este salto de

13

qualidade. Seguem ento alguns exemplos concretos: a Nos ultimos anos, dentre os medalhistas em olimp adas cient cas (matemtica, f a sica, qu mica,...) de n vel regional e nacional, h sempre presena marcante de cearenses. a c Nos concursos e vestibulares mais concorridos do pa como por exemplo o ITA e o IME, a quantidade de s, estudantes de Fortaleza aprovados vem crescendo a cada ano. Para se ter uma idia, no ultimo vestibular do e ITA (2003-2004), s a cidade de Fortaleza aprovou uma quantidade de alunos comparvel (salvo engano maior) o a a todo o estado de So Paulo. a Agora uma estat stica curiosa: nos ultimos anos, em duas oportunidades (1994 e 2000) o primeiro lugar do vestibular para Medicina na UFC, foi um estudante que havia participado da Olimp ada Internacional de Matemtica. Em 1994, o estudante Marcondes Frana foi a IMO em Hong Kong e em 2000 o estudante Ulisses a c Medeiros foi a IMO na Coria. Ambos amigos de muito bom grado, hoje com bastante sucesso em suas carreiras e mdicas. e Resumindo, um diretor que implante essas idias em sua escola pode esperar a mdio prazo um salto de qualidade na e e formaao de seus alunos, que ser o reexo do desempenho deles nas competioes culturais e vestibulares. c a c Esses resultados se tornam com freqncia, e merecidamente, motivo de marketing das escolas particulares de Fortue aleza. E agora tambm do Governo do Estado, que atravs do Projeto Numeratizar iniciou no ano passado (2003) e e uma Olimp ada de Matemtica da Rede Pblica que reuniu 150.000 alunos, e tem tudo para comear a reerguer o a u c ensino pblico do nosso Estado. u

2.3

Que professores/alunos devem participar?

A princ pio, todos os professores de matemtica interessados podem fazer parte do novo projeto de olimp a ada da escola. Pode ser feita uma reformulaao do horrio de cada um, por exemplo um professor que tem 20 horas semanais c a em uma escola pode acertar com a direao para ter 16 horas em sala de aula usual e 4 horas com turmas de olimp c ada. Denidos aqueles professores que vo dar aulas para a preparaao de alunos para a olimp a c ada, necessrio que o e a material a ser abordado seja dividido entre eles (discutiremos sobre o material mais adiante) e que seja combinado quem vai ser responsvel por qual turma e em que horrio (j falaremos disso tambm). a a a e E os alunos? Como devem ser escolhidos? Para uma escola que j possui um projeto como este, ou escolas com a grande quantidade de alunos, geralmente feito um pequeno teste de seleao. Esta deve ser uma prova apenas para e c detectar os poss veis talentos que voc professor vai ter em suas turmas. e O ideal que seja permitida a participaao de todos os alunos interessados, sem cortes iniciais. Pois a experincia e c e nos diz que uma turma de olimp ada que comea por exemplo com 30 alunos, depois de um semestre tem apenas 20, c depois de um ano, 15 ou menos. H uma evaso natural dos alunos. Por vrios motivos: a a a Nem todo mundo vai querer car indo para a escola em horrios extras. Muitos vo olhar logo e dizer: a a -T fora! J sou muit o a ssimo ocupado (isso o que ele pensa). Isso coisa de nerd... e e Outros comeam e no aguentam o rojo. Tenha em mente que nem todos que esto l tero vocaao para c a a a a a c matemtica (certamente tero para outras matrias...). Alguns desses se esforaro e aprendero muitas coisas, a a e c a a outros desistiro. No que triste. a a Aviso aos professores: voc ter consigo vrios tipos de aluno. Sua turma pode ser bastante heterognea inicialmente, e a a e mas no se preocupe, isso se resolver com o tempo. Vale muito mais a pena trabalhar com alunos esforados e a a c estudiosos do que com aqueles que se acham superiores e no so nada modestos (estes podem at ser realmente a a e inteligentes...), mas que s cam atrapalhando a aula. Ou seja, ponha logo tudo nos eixos e dite as regras: que ali o ningum melhor do que ningum. e e e

14

2.4

Como denir as turmas e os horrios das aulas a

A Olimp ada de Matemtica vem se tornando bastante competitiva ao longo dos anos. A cada ano mais pessoas vm a e participando, e como resultado disto a seleao cada vez maior. c e No apenas por este motivo, o ideal que um aluno comece a se preparar para fazer Olimp a e ada de Matemtica a o mais cedo poss vel. Quando o aluno mais novo, sua base ainda est em formaao, e desta forma a preparaao e a c c ol mpica pode ajudar bastante no desenvolvimento de sua maturidade matemtica. Alm disso tem-se mais tempo a e para trabalhar com o(a) garoto(a). Quando pensamos em preparar um aluno j mais velho (digamos 2o ou 3o ano do ensino mdio), devemos corria e gir vrios defeitos de base que ele j adquiriu ao longo da vida, para depois comear a ensinar alguns tpicos de a a c o matemtica ol a mpica e a desenvolver as idias. E ento, qunado voc v que o aluno tem potencial, ele termina o 3o e a e e ano, e no vai mais participar das competioes sob sua tutela. a c E portanto fortemente recomendado que voc comece o seu projeto de Olimp e ada de Matemtica com turmas de a 5a srie at o 1o ano. Se poss uma turma para cada srie. Caso isso no seja poss e e vel e a vel, um outro modo seria dividir os alunos pelos n veis da OBM: n I (5a e 6a sries), n II (7a e 8a sries) e n III (ensino mdio). vel e vel e vel e As aulas de olimp ada devero ser em horrios extra-classe, ou seja em per a a odos diferentes das aulas usuais dos alunos. Por exemplo se os alunos da escola assitem aula pela manh, voc pode marcar as aulas de olimp a e ada duas vezes por semana ` tarde de 2:30 as 4:10. Ou se voc tem alunos em dois turnos, manh e tarde, e quer reuni-los a e a numa s turma de olimp o ada, isso pode ser feito duas vezes por semana ` noite (6:30 - 8:00), ou aos sbados. a a De fato, os horrios vo se acomodar de acordo com a necessidade e disponibilidade dos alunos e professores ena a volvidos. Agora quantas aulas deve ter cada turma de olimp ada? Bem essa uma pergunta ex e vel. Acho que em 4 horas semanais (2 aulas de 2 horas) para cada turma j se pode desenvolver um bom trabalho. Se o seu colgio j a e a vem com uma turma que passou pela 5a, 6a, 7a e agora est na 8a srie, voc pode pensar em aumentar para 6 horas a e e semanais (isso vale para turmas experientes). Caso a situaao esteja economicamente dif c cil, tambm podemos pensar em 4 horas semanais por n e vel. O que daria um custo inicial para a escola de 8 aulas se pensarmos em comear s com n c o veis I e II. Agora, o importante comear!! E bater o centro. Depois as coisas vo se acertando, voc pe mais umas aulas ali, d um intensivo e c a e o a a perto das provas, faz um cursinho nas frias... e Outra dica importante. Se voc for optar por dar 4 ou 6 aulas em uma turma, de preferncia faa isso em dias e e c intercalados (seg-qua-sex ou ter-qui ou qua-sb). Intercalando os dias eles vo ter mais contato com voc, alm do a a e e que, voc pode explorar mais o enorme potencial que tm os exerc e e cios pra casa, e pass-los duas ou trs vezes por a e semana. E claro, como j disse antes, isso vai depender da disponibilidade de tempo/dinheiro de todos. a

2.5

Material necessrio a

Agora chegamos a um ponto crucial. Voc, professor deve tomar alguns livros e materiais como base para treinar seus e alunos. Surpreenda-se com as coisas novas que voc mesmo vai aprender! e A maior fonte de material e sabedoria dos tempos modernos chama-se Internet. H diversos sites que disponibia lizam muito material para o treinamento ol mpico. E o mais importante: de graa. Basta entrar num site de busca e c se aventurar. Abaixo falaremos um pouco dos tpicos a serem estudados para a Olimp o ada de Matemtica, os livros recomena dados para iniciantes e alguns sites interessantes. Tradicionalmente, os assuntos abordados so divididos em 4 temas: Teoria dos Nmeros, Algebra, Geometria e a u Combinatria. Falaremos um pouco de cada um deles, indicando alguns livros e ainda apresentando uma sugesto de o a contedo a ser trabalhado. u

15

2.5.1

Teoria dos N meros u

Os tpicos em Teoria dos Nmeros so aqueles relacionados com as propriedades elementares dos nmeros inteiros: o u a u divisibilidade, nmeros primos, fatoraao, MDC, MMC,... u c Este um tpico clssico na olimp e o a ada, e fortemente recomendado para iniciar o treinamento de jovens, pois vrios a destes assuntos j so abordados no colgio. a a e Livros recomendados para iniciantes: Teoria Elementar dos Nmeros, Edgar de Alencar Filho (Brasil) u Aritmetica, Maria Elena Becker, Ed. Red Olimpica (Argentina) Aritmetica, Saulo Aranda (Venezuela) Livro recomendado para n Intermedirio: vel a Introduao ` Teoria dos Nmeros, Jos Pl c a u e nio Santos, Coleao Matemtica Universitria (Brasil) c a a No obrigado o estudo por um destes livros acima, qualquer livro que cubra os mesmos assuntos pode ser utilizado a e como base.

2.5.2

Algebra

Nesta rea, os alunos devem aprender bem as operaoes com nmeros e suas propriedades. A lgebra estuda princia c u a palmente as equaoes, de primeiro e segundo graus, os sistemas de equaoes, os produtos notveis. Para um aluno do c c a ensino mdio, quando falamos em lgebra, englobamos diversos assuntos: polinmios, nmeros complexos, seqncias... e a o u ue A lgebra bastante ensinada no colgio. Da o primeiro passo do aluno ser compreender bem as lioes da esa e e , a c cola, para depois vir a se aprofundar nas aulas de olimp ada. Livro recomendado para iniciantes: Livros do colgio em geral. e Livro recomendado para n intermedirio: vel a Challenging Problems in Algebra, I.M. Yaglom (EUA) Veja mais sobre lgebra na seao: Tcnicas de Resoluao de Problemas. a c e c

2.5.3

Geometria

Aqui todos ns sabemos do que se trata. E a clssica geometria plana. Sempre presente nas provas de olimp o a ada. Tambm bastante ensinada em sala de aula, de modo que o aluno deve saber bem a matria do colgio para depois e e e e se aprofundar. Livro recomendado para iniciantes: Matemtica Elementar, vol. 9, Gelson Iezzi et al. (Brasil) a Livros recomendados para n intermedirio: vel a Geometria, vols 1 e 2 , A.C. Morgado (Brasil) Challengig Problems in Geometry , I.M. Yaglom (EUA) Geometry Revisited, H.S. Coxeter (EUA)

16

2.5.4

Combinatria o

D-se o nome de combinatria a tudo aquilo que no teoria dos nmeros, lgebra ou geometria. Os problemas de a o a e u a combinatria so aqueles leg o a timos de olimp ada, que usam bastante a criatividade do aluno. Na lista de tpicos a o serem estudados merecem destaque: paridade, contagem, princ pio das gavetas, jogos, invariantes... Este assunto a grande novidade. Geralmente os estudantes no tm contato com este tipo de matemtica na e a e a escola, por isso os professores devem ir bem devagar e sempre estimulando o racioc nio dos seus alunos. Livros recomendados para iniciantes: Mathematical Circles, the russian experience, D. Fomin (Russia - traduzido para o ingls) e Contos com contas, Miguel de Guzman (Portugal) Aventuras Matemticas, M. Guzman (Portugal) a Livro recomendado para n intermedirio vel a Combinatria e Probabilidade, A.C. Morgado et al (Brasil) o

2.5.5

Tcnicas de Resoluo de Problemas e ca

Os alunos que se preparam para as olimp adas de matemtica devem ter seu treinamento voltado para a resoluao de a c problemas. Um grande amigo, Rui Lopes Viana Filho, medalha de ouro na Olimp ada Internacional de Matemtica a em 1998, costumava dizer que seu treinamento matemtico era baseado na resoluao de exerc a c cios. Havia o que ele chamava de sua caixa de ferramentas que eram aqueles problemas-chave que ele j havia resolvido. Ento quando a a era chegada a hora das provas, de resolver novos problemas, ele sempre recorria ` sua caixa de ferramentas para ver a se encontrava alguma idia parecida. E mais ou menos assim que as coisas funcionam. e Alguns livros recomendados para o Olimp ada de Matemtica so coletnias de problemas e soluoes. Estes abordam a a a c todos os assuntos supracitados e so de fundamental importncia para os professores (que podem fazer listas, simua a lados, aulas de problemas...) e para os alunos (que podem tentar resolv-los e caso no consigam, podem aprender e a novas tcnicas entendendo as soluoes). e c Abaixo listamos vrios destes livros. a Livros recomendados para iniciantes: Olimp adas de Matemtica do Estado do Rio de Janeiro, Eduardo Wagner et al (Brasil) a Olimp adas Paulistas de Matemtica, 1o grau (Brasil) a Olimp adas Paulistas de Matemtica , 2o grau (Brasil) a Contest Problem Book (AHSME e AIME), volumes 1 a 6 (EUA) Livros recomendados para n intermedirio: vel a The Hungarian Problem Book I, II, III (Hungria) Olimp adas Brasileiras de Matemtica 1a a 8a (Brasil) a Olimp adas Brasileiras de Matemtica 9a a 16a (Brasil) a Olimp adas de Matemtica do Cone Sul, 1a a 4a (Argentina) a Olimp adas de Matemtica do Cone Sul , 5a a 12a (Brasil) a Olimp adas Iberoamericanas de Matemtica 1a a 10a (Brasil) a

17

2.5.6

Revista Eureka!

Em 1998, com a reestruturaao da Olimp c ada Brasileira de Matemtica, foi criada a revista Eureka!. Esta uma a e publicaao trimestral, que atualmente est em seu volume 19, cujo principal objetivo o treinamento brasileiro para c a e as olimp adas de matemtica. Esta revista extremamente recomendada para um projeto de olimp a e ada na escola. Para saber mais informaoes de como assinar e como pedir os nmeros anteriores consulte o site ocial da OBM: c u www.obm.org.br

2.5.7

Livros para estudos avanados c

Abaixo listamos mais alguns livros, agora mais avanados, voltados para o treinamento ol c mpico. Estes so recomena dados apenas para alunos do ensino mdio que j vm em preparaao constante e para professores. Todos so escritos e a e c a em ingls. e 1. Olimp adas Internacionais de Matemtica 1959-1974 (EUA) a 2. Olimp adas Internacionais de Matemtica, 1975-1980 (EUA) a 3. Olimp adas Internacionais de Matemtica, 1981 - 1994 ( a India) 4. Principles and Tecniches in Combinatorics, Cheng C. Chong (Taiwan) 5. Winning Solutions , Cecil Russeau and E. Lozanski (EUA) 6. 102 Problems in Combinatorics , Titu Andreescu (EUA) 7. Mathematical Miniatures , T. Andreescu e S. Savchev (EUA) 8. Mathematical Contests, todos os 7 volumes (95, 95-96, 96-97, 97-98, 98-99, 99-00, 00-01) 9. 101 Problems in Algebra, T. Andreescu (Australia) 10. Advanced Euclidean Geometry, M. Johnson (EUA) 11. Geometry Revisited, H.S. Coxeter et al (EUA) 12. USA Mathematical Olympiads, M. Klamkim (EUA) 13. The Tournament of the Towns, todos os volumes (Australia) 14. Combinatorics, Ioan Tomescu (Romenia) 15. Problems in Combinatorics and Graph Theory, I. Tomescu (Romenia) 16. Polynomials, E. Barbeau (EUA) 17. Complex Numbers in Geometry , S. Zahn (Coria) e 18. Mathematical Gems, volumes 1,2,3. H. Honsberger (EUA) 19. Mathematical Morsels, H. Honsberger (EUA) 20. Mathematical Mosaics , R. Vakil (Canad) a 21. In Plyas Footsteps, (EUA) o 22. Problem solving through problems, I. Larson (EUA) 23. Problem Solving Strategies, A. Engel (Alemanha) 24. Geometric Inequalities , O. Botemma (EUA) 25. The Putnam Competition, volumes 1,2,3 (EUA) 26. Introduction to the Theory of Numbers, Niven and Zuckerman (EUA) 27. Elementary Number Theory, E. Burton (EUA)

18

2.5.8

Buscando na Internet

Abaixo esto listados alguns sites bastante interessantes. Nestes sites voc pode encontrar informaoes sobre a a e c olimp ada de matemtica, provas de anos anteriores, material de treinamento, revistas, fotos e vrias outras coisas. a a A Internet uma fonte inesgotvel de conhecimento e informaao. Certamente os alunos vo se empolgar muito e a c a mais se puderem ver fotos, listas de medalhados, curiosidades... E se eles prprios puderem fazer download de provas o e material de treinamento, melhor ainda! www.obm.org.br - a pgina ocial da Olimp e a ada Brasileira de Matemtica. Contm tudo que voc precisa a e e saber e ainda com inmeros links para os sites ociais das Olimp u adas Estaduais. www.sbm.org.br - a pgina ocial da Sociedade Brasileira de Matemtica, onde voc pode se cadastrar e e a a e ganhar inmeras vantagens como por exemplo descontos na compra de livros. u www.oma.org.ar - pgina da Olimp a ada de Matemtica Argentina. a www.numeratizar.mat.br - pgina do projeto Numeratizar, que a Olimp a e ada da Escola Pblica do Cear, u a com material de treinamento elementar. www.opm.mat.br www.impa.br - Olimp ada Paulista com vrias coisas legais a - Grupo Teorema de Matemtica, tambm com inmeras coisas legais. a e u

www.teorema.mat.br

- Site do IMPA - Instituto de Matemtica Pura e Aplicada. a - Olimp ada Peruana de Matemtica (esse link est na OBM) a a

http://www.geocities.com/CollegePark/7174 www.google.com.br desejem.

- site de busca: deve ser o mais utilizado por vocs para procurar qualquer coisa que e

2.6

Sugesto de programas a

No Apndice A do livro, damos sugestes de programas para o treinamento ol e o mpico. Os professores devem adequar estas sugestes ` sua disponibilidade de tempo, de livros e ao rendimento dos seus alunos. o a Os contedos vo desde a 4a srie at o 2o ano do ensino mdio. Era um pouco grande para ser inclu aqui, u a e e e do no meio do texto, por isso, resolvi deixar como anexo.

2.7

Posso sofrer rejeio de alunos/professores da escola? ca

Toda novidade traz consigo admiraao de uns, rejeiao de outros. So os dois lados da moeda. Experincias comproc c a e vam que pode haver certa rejeiao ao programa de treinamento ol c mpico na sua escola. Tambm no h motivo para e a a desespero, pois so situaoes perfeitamente contornveis. a c a Certas facoes de alunos podem no querer participar do seu projeto. Natural. Opao deles. Se eles insistirem c a c em tratar seus alunos como CDF, bitolados, nerds... no se preocupe, isso normal e vai passar com o tempo. Em a e certos estados, isso reete o pensamento da juventude em geral, de que ser estudioso e esforado errado, coisa de c e e extraterrestre, enquanto o legal ser relaxado e passar se arrastando, mas ser um cara da galera. e Fico triste com tais atitudes, mas ainda assim acredito que esta mentalidade v mudar ao longo do tempo. Em a meu estado, o Cear, os alunos esforados e inteligentes, so muito estimados por seus colegas e por suas escolas, a c a alguns at com certas regalias. e O que mais me preocupa a rejeiao que pode haver por parte da direao e de outros professores da escola. Voc pode e c c e se perguntar: Ora por que essas pessoas vo querer atrapalhar? a Eu diria que comodismo ou inveja da parte delas. So aquelas pessoas que no querem mudar, que acham que e a a

19

tudo est bom como est, e quando aparece uma pessoa propondo uma novidade para desenvolver as coisas, elas cam a a desencorajando: isso no vai dar certo, melhor nem comear,... a c O que fazer com essas pessoas? Bem, o melhor dizer: se voc no quer ajudar, tudo bem, mas por favor tambm no e e a e a atrapalhe! Desista de convencer todo mundo que a sua idia boa. Voc no vai conseguir. S comeando e passando e e e a o c algum tempo que voc poder mostrar, atravs do seu trabalho, como aquilo valeu a pena. Acima de tudo: no se e e a e a preocupe, essas vozes opositoras vo caindo ao longo do tempo. a Apie e d crdito tambm aos outros projetos da escola: caminhada ecolgica, grupo de dana, grupo de teatro, o e e e o c ONGs,... pois assim como o seu, todos eles so importantes! Faa como acima, se no quiser ajudar, tambm no a c a e a atrapalhe. Fique de olho tambm na parte dos educadores que dizem que a Olimp e ada de Matemtica um processo exclua e dente. NAO E! Onde j se viu tamanha tolice, me perdoem as palavras! E claro que alguns alunos vo se dar melhor a a que outros, mas isso muito natural. Obviamente estes outros sero muito bons em outras reas, talvez artes, msica, e a a u esportes... basta que lhes sejam dadas as oportunidades. E isso que a Olimp ada de Matemtica, entre outras coisas, a faz: d oportunidade aos alunos para desenvolverem seu potencial. a Os educadores que pensam na Olimp ada de Matemtica como fator de diferenciaao social e excluso so os mesmos a c a a que desejam que os alunos sejam tratados como iguais: sem oportunidade e nem perspecticas para nenhum. Vamos mudar esse quadro. Vamos dar chances, sonhos e rumos a quem quiser segui-los. Que bom se isso pudesse ser feito em todos os ramos, no s em matemtica! a o a

2.8

O que eu (professor) ganharei com isso?

Os professores que participarem do projeto de Olimp ada de Matemtica da escola s tm a ganhar. Um grande a o e presente(!?) que eu considero a mudana da sua rotina, pois agora voc vai ter que preparar novas aulas, com n e c e vel de diculdade maior do que o da sala de aula comum. Certamente ser mais dif no comeo, mas podem ter certeza que muito recompensador. As aulas de olimp a cil c e ada no precisam ser to formais e r a a gidas como as aulas do horrio de classe. Com isso, o professor se aproxima mais a dos alunos e se torna realmente amigo deles. Ao longo do tempo a aula vai se tornando cada vez mais um bate-papo divertido entre amantes de matemtica. a Outro bnus que eu aponto que voc, professor, vai enriquecer ainda mais seu conhecimento matemtico. Ao o e e a preparar e ministrar sua aulas de olimp ada, certamente voc vai se deparar com coisas novas, e vai tambm aprender e e bastante. Todos ns temos algo a aprender: os alunos conosco e ns com eles. o o

2.9

O que meus alunos ganharo com isso? a

Os alunos que participarem tero ainda mais recompensas. J disse anteriormente que a olimp a a ada far com que o a aluno estimule seu racioc nio e isso vai possibilitar um desenvolvimento educacional melhor, no s em matemtica, a o a como tambm nas outras reas. e a Alm disso, na olimp e ada se faz muitos amigos. Pessoas de outras escolas, at de outros estados, com interesses e parecidos. Se algum deles tiver o prazer de se classicar na OBM, por exemplo, e vir a participar de um encontro como a Semana Ol mpica, ser uma experincia inesquec para o garoto. Garanto que ele ser estimulado em 100% a e vel a para continuar estudando. A Olimp ada de Matemtica tende a crescer no pa e isto passa a ser tambm uma carta de recomendaao para a s, e c o estudante adiante. Premiaoes na Olimp c ada sero importantes diferenciais na vida acadmica, quando o aluno a e for concorrer a bolsas na universidade. No s a bolsas, mas tambm a qualquer tipo de emprego. Um diploma de a o e olimp ada, assim como um diploma de curso de ingls ou informtica, passa a fazer parte do curr e a culo do aluno.

20

Aqui no Cear, este ano, os 363 alunos classicados na Olimp a ada de Matemtica da Escola Pblica esto recebendo a u a bolsas mensais no valor de R$72, 00. J pensou o quanto isso signica para fam a lias carentes? E o governo tomando uma iniciativa pioneira, louvvel, de premiar quem estuda. Esses alunos esto ainda recebendo treinamento em matemtica a a a e certamente no futuro difundiro seus conhecimentos e estaro aqui para falar dos benef a a cios que a matemtica lhes a trouxe.

2.10

Montando uma mini-biblioteca

Com o passar do tempo e armaao do projeto de olimp c ada na escola, importante a implantaao de uma minibibe c lioteca de matemtica. Esta pode ser apenas uma estante na biblioteca da escola, ou um armrio na sala do professor, a a no importa. a O que importa que livros como os que foram citados acima (e por que no tambm o material didtico usado e a e a nas aulas?) sejam disponibilizados para os alunos. A escola no precisa adquirir todos os livros de uma vez, basta a comprar 1 ou 2 por ms, que em 3 anos sua minibiblioteca vai estar excelente! e O acesso a ela pode ser apenas `s pessoas interessadas na olimp a ada, atravs de carteirinha, chave do armrio, etc, e a ou se preferirem a qualquer pessoa do colgio. Tambm no precisa dizer que um cuidado com os livros essencial. e e a e O rendimento dos alunos vai melhorar consideravelmente se eles puderem, de vez em quando levar um livrinho para casa...

2.11

Contratempos que podem acontecer

Tudo bem. Voc j iniciou seu projeto, tm alunos ass e a e duos em vrias turmas de olimp a ada, competindo a n vel municipal, estadual e nacional. J venceu a rejeiao inicial de alguns alunos e cabeas da direao. O que pode dar de a c c c errado agora? Eu diria que sua base j est slida. Agora s tocar pra frente, tomando cuidado com poss a a o e o veis contratempos. Vou chamar a atenao para dois poss c veis acontecimentos. E natural que voc tenha turmas melhores que outras, mas nem por isso concentre toda a sua atenao em uma e c turma s. Esteja sempre olhando a formaao dos alunos que vm a seguir. E importante o uxo cont o c e nuo, ou seja, formar bem os alunos da 8a srie, os da 7a, os da 6a, os da 5a. Pois no outro ano tudo recomea. e c Ocorreu na escola onde trabalhei aqui em Fortaleza. T nhamos uma turma muito boa, com uns 10 alunos excelentes, diversas vezes premiados. Todos os professores queriam dar aula para esses caras e meio que esqueciam os demais. Resultado: quando essas turma concluiu o colgio, havia s 1 estudante de olimp e o ada na turma da srie e anterior. Fez-se um hiato, que s conseguimos reparar 1 ou dois anos depois. Desde ento trabalhamos com atenao o a c para com todas as sries. e Outro aspecto relevante sempre conscientizar seus alunos que a matemtica que eles aprendem na olimp e a ada e diferente da matemtica da sala de aula. Dizer aos alunos que eles no devem esnobar colegas ou professores. Hua a mildade sempre. Na maioria dos casos, os alunos nem pensam em esnobar algum (a no ser seu irmo mais novo...), e a a mas de vez em quando aparecem uns espertinhos, a voc deve baixar a bola deles. e

2.12

Como dar continuidade ao processo?

J foi dito algo sobre isso anteriormente. Sobre a atenao para com todas as turmas, para que no haja buracos. a c a Sobre a minibiblioteca. Esses so alguns fatores importantes na continuidade do processo. a E aconselhvel que os professores se renam de vez em quando, para avaliar o progresso dos seus estudantes e os a u rumos a serem tomados. Diviso dos contedos entre os professores tambm muito importante. Para que cada um a u e e

21

saiba suas obrigaoes e seus objetivos. c Outros tpicos relevantes para dar prosseguimento ao seu projeto ol o mpico so: o est a mulo constante aos seus alunos e professores, e tambm o contato com a SBM, OBM e universidades. Estes sero discutidos em separado nas seoes e a c seguintes.

2.13

Est mulo constante aos seus alunos

Chegamos a um tpico crucial. Como manter seus alunos estimulados? o O prprio clima de amizade presente na aula de olimp o ada j um bom est a e mulo para os garotos. Vale tambm e de vez em quando distribuir uns prmios, fazer umas gincanas na sala de aula, sempre promovendo a competiao e c matemtica saudvel entre eles. a a A minibiblioteca e o acesso ` Internet so sem dvida armas poderosas no est a a u mulo a seus alunos. Atravs destas e o estudante se sentir cada vez mais seguro para pesquisar e com isso passar rapidamente a desenvolver seu cona a hecimento matemtico por conta prpria! Com o passar do tempo, eles (os alunos) vo comear a participar das a o a c competioes de matemtica. A OBM e a Olimp c a ada Estadual talvez sejam dif ceis no comeo e provvel que voce c e a no veja seus alunos se classicarem na primeira vez. Nada de desnimo! Uma boa conversa com eles, para que eles a a possam assimilar o revs, vai cair bem. e Seria interessante que a escola organizasse uma olimp ada interna, fazendo uma festinha e congregando pais, alunos e professores. Distribua vrias medalhas e prmios, assim os garotos vo car felizes...(seus pais tambm!!!). a e a e Voc pode tambm conversar com outros professores da sua cidade, e montar uma olimp e e ada do seu munic pio. Competindo na esfera municipal, todos os alunos tero mais chances de premiaao. a c Trabalhando nessas competioes menores, eles vo se sentir cada vez mais seguros e capazes para concorrer a n c a vel estadual e nacional. E o mais importante, estudaro sempre com mais dedicaao. Assim voc estar no caminho certo. a c e a

2.14

Estreitando os laos com SBM e as universidades c

E bastante recomendado que o professor se associe ` SBM, pois isso garante uma srie de vantagens: descontos para a e comprar livros, para receber a Revista do Professor de Matemtica, para participar de eventos. A SBM, atravs da a e OBM a principal apoiadora da Olimp e ada de Matemtica no nosso pa estimulando os alunos atravs de listas de a s, e discusso via internet e da revista Eureka. E tambm a SBM/OBM que contribui com parte da verba destinada para a e a realizaao das competioes regionais, que vm crescendo ao longo do tempo. Hoje em dia, a maioria dos estados c c e brasileiros tm sua prpria Olimp e o ada de Matemtica. a Alm disso, o contato com a Universidade tambm deve ser estreito, pois no esqueamos que esta deve ser o objetivo e e a c para os nossos alunos. E importante que os professores do ensino mdio e fundamental possam trocar id com os e eas mestres e doutores do ensino superior e ter acesso `s bibliotecas (isso legal!!). Com o passar do tempo, a universidade a e pode criar programas de vero para jovens interessados na olimp a ada, como j feito no Rio e no Cear, por exemplo. ae a H jovens talentos que, no IMPA(RJ), iniciam seus estudos de mestrado, e em seguida doutorado, ainda adolescentes a e se tornam excelentes matemticos. No tenha dvida que a universidade vai dar muito apoio a esses jovens talentos, a a u e eles s tero a agradecer. o a

22

Cap tulo 3

Listas de Exerc cios


Este cap tulo foi idealizado com o objetivo de mostrar mais alguns problemas do interessante mundo novo da Matemtica. a So 10 listas, cada uma com 5 problemas, recomendadas para alunos e professores iniciantes. Voc pode us-las como a e a bem entender com seus pupilos. Uma dica: passe uma lista como essas (4 ou 5 problemas) uma vez por semana para eles. Estes so problemas que cobrem os mais variados tpicos de matemtica para um aluno iniciante. Existe uma certa a o a ordem de diculdade nas listas. Voc ver que os problemas se tornam cada vez mais dif e a ceis (mas s um pouquinho...). o Boa sorte!

Lista 1
Problema 1 Colocar em cada um dos pontos marcados sobre os lados do tringulo os nmeros de 1 a 9, de modo que a soma dos a u quatro nmeros colocados sobre cada lado seja a mesma. u

Problema 2 Um pai tem 40 anos e a soma das idades de seus trs lhos 22 anos. Dentro de quantos anos a idade do pai ser a e e a soma das idades dos lhos? Problema 3 Em um ano no bissexto, qual o dia do meio? (Isto , aquele cujo nmero de dias anteriores a ele igual ao nmero a e e u e u de dias posteriores a ele) Problema 4 Temos trs caixas (em la) e trs frutas: um abacaxi, uma banana e um caj. Cada caixa contm uma fruta e sabe-se e e a e que: A caixa verde est ` esquerda da caixa azul. aa A caixa vermelha est ` direita da banana. aa 23

24

O caj est ` direita da caixa vermelha. a aa Qual fruta est em qual caixa? a Problema 5 Raul e Cida formam um estranho casal. Raul mente `s 4as, 5as e 6as feiras, dizendo a verdade no resto da semana. a Cida mente aos domingos, 2as e 3as feiras, dizendo a verdade nos outro dias. Certo dia ambos declaram: amanh ae dia de mentir. Em que dia foi feita esta declaraao? (Olimp c ada Carioca/93)

Lista 2
Problema 6 Bruce Willis se meteu numa tremenda enrascada: est em uma fonte, na qual est armada uma bomba que vai explodir a a em alguns minutos. Se ele tentar sair da fonte, tudo vai pelos ares... mas ele tem uma chance: conectada ` bomba a existe uma balana, e esta balana desarma a bomba quando se pem exatamente 4 litros de gua sobre ela. Bruce c c o a Willis tem 2 recipientes: um com capacidade para 3 litros e outro com capacidade para 5 litros, como ele deve fazer para sair vivo dessa fria? (baseado numa cena do lme Duro de matar III). Problema 7 Bruno falou: Antes de ontem eu tinha 10 anos, mas completarei 13 ano que vem. E poss vel que Bruno esteja falando a verdade? Problema 8 Augusto possui uma grande quantidade de 0 s, 1 s, 3 s, 4 s, 5 s, 6 s, 7 s, 8 s e 9 s, mas dispe somente de vinte e dois o 2 s. At que pgina ele poder numerar as pginas de seu novo livro? (Olimp e a a a ada Carioca - 94) Problema 9 Um crime cometido por uma pessoa e h quatro suspeitos: Andr, Eduardo, Rafael e Joo. Interrogados, eles fazem e a e a as seguintes declaraoes: c 1. Andr: Eduardo o culpado. e e 2. Eduardo: Joo o culpado. a e 3. Rafael: Eu no sou culpado. a 4. Joo: Eduardo mente quando diz que eu sou culpado. a Sabendo que apenas um dos quatro disse a verdade, quem o culpado? e Problema 10 (i) Mostre como dispor os nmeros 1, 2, 3, 4, 5, 6, 7, 8, 9 nas casas de um quadrado 3 3 para formar um quadrado u mgico, ou seja, para que a soma dos nmeros nas linhas, colunas e diagonais seja a mesma. a u (ii) Prove que em todo quadrado mgico o nmero da casa central deve ser o 5. a u (iii) Voc seria capaz de pr os nmeros de 1 a 16 (sem repetiao) em um quadrado 4 4 de modo a formar um e o u c quadrado mgico? Lembre-se que a soma dos nmeros em cada linha, coluna e nas duas diagonais deve ser igual. a u

25

Dica absurdamente boa: Tente primeiro mostrar que a soma comum (de cada linha, coluna ou diagonal) deve ser 34.

Lista 3
Problema 11 Sobre uma mesa h 21 garrafas de guaran, das quais 7 esto cheias, 7 esto vazias e 7 esto cheias at a metade. a a a a a e Reparta-as entre trs garotos de modo que os trs recebam a mesma quantidade de garrafas e de guaran. e e a Obs: No vale derramar o guaran de uma garrafa em outra. a a Problema 12 No barril A h 100 litros de gua e no barril B h 100 litros de lcool. Retira-se 1 litro de gua de A e coloca-se em a a a a a B. Em seguida retira-se 1 litro da mistura de B e coloca-se em A. No nal das operaoes, h mais gua no lcool ou c a a a mais lcool na gua? a a Problema 13 Chamam-se pal ndromos os nmeros inteiros positivos que no se alteram quando invertida a ordem de seus algarismos u a e (por exemplo: 383, 4224, 74847...). Qual o nmero total de pal e u ndromos de cinco algarismos? Problema 14 (OBM/98) Pintam-se de preto todas as faces de um cubo de madeira cujas arestas medem 10 cent metros. Por cortes paralelos `s faces, o cubo dividido em 1000 cubos pequenos, cada um com arestas medindo 1 cent a e metro. Determine: (a) o nmero de cubinhos que no possuem nenhuma face pintada de preto. u a (b) o nmero de cubinhos que possuem uma unica face pintada de preto. u (c) o nmero de cubinhos que possuem exatamente duas faces pintadas de preto. u (d) o nmero de cubinhos que possuem trs faces pintadas de preto. u e Problema 15 Considere a sequncia (1, 2, 2, 3, 3, 3, 4, 4, 4, 4, 5, 5, 5, 5, 5...), cujos termos so os inteiros consecutivos em ordem crese a cente, e na qual o inteiro n ocorre n vezes. Ache o resto da diviso por 5 do 1999 termo. (Olimp a ada Carioca 93)

Lista 4
Problema 16 Escrevemos a lista dos nmeros naturais 1, 2, 3, 4, 5, 6, 7, 8, 9, 10, 11, 12, 13, .... Qual o d u e gito que ocupa o 2004 lugar? Problema 17 Na soma IRA + ARO + ORA, as letras I, R, O, A representam os d gitos 1, 3, 8 e 9 (d gitos distintos para letras distintas, no necessariamente nessa ordem). a (a) Qual a maior soma poss e vel? (b) E a menor? Problema 18

26

(a) Uma pea colocada em uma extremidade (do lado de fora) de uma la 1 21 de casas. Cada jogador na sua c e vez pode mover a pea uma ou duas casas para a frente (so dois jogadores). Ganha o jogador que atingir a c a ultima casa do tabuleiro. Quem possui a estratgia vencedora? e (b) Generalize para o caso de o tabuleiro ser 1 N . Problema 19 (Rioplatense/2003) Algumas pessoas vo a uma pizzaria. Cada pessoa que est faminta quer comer 6 ou 7 pedaos a a c de pizza, e as outras querem comer 2 ou 3 pedaos. Cada pizza possui 12 pedaos. Sabe-se que 4 pizzas no so c c a a sucientes para satisfazer a todos, mas com 5 pizzas sobrariam alguns pedaos. Quantas pessoas foram ` pizzaria? c a Quantas delas estavam famintas? Problema 20 Temos uma cartolina em forma de tringulo equiltero. a a (a) Mostre como dividir a cartolina em 22 tringulos equilteros menores (no necessariamente do mesmo tamanho). a a a (b) Mostre como dividir a cartolina em 17 tringulos equilteros menores (no necessariamente todos do mesmo a a a tamanho).

Lista 5
Problema 21 Quando 1094 94 desenvolvido, qual a soma de seus algarismos? e e Problema 22 Daniel trocou de lugar as teclas da calculadora de Onofre da seguinte maneira: Calculadora de Onofre 7 4 1 8 5 2 0 9 6 3 Calculadora com as teclas trocadas 3 2 1 6 9 5 8 4 7 0

Sem notar que as teclas foram trocadas, Onofre multiplica dois nmeros de dois d u gitos cada um e aparece como resultado 1722. Sabe-se que o verdadeiro resultado seria um nmero de trs algarismos. Que nmeros Onofre queria u e u multiplicar ? Problema 23 Em um tringulo issceles ABC (AB = AC, BAC = 30 ) marcamos um ponto Q no lado AB e um ponto P na a o mediana AD, de modo que P C = P Q (Q = B). Ache P QC. Problema 24 (Bielorssia/01) Encontre o menor nmero poss de elementos do conjunto: u u vel {1, 2, 3, ..., 28} que devem ser deletados para que o produto dos restantes seja um quadrado perfeito.

27

Problema 25 Vanessa escreve oito nmeros em cada um dos vrtices de um cubo. Em cada aresta escreve a soma dos dois nmeros u e u dos vrtices adjacentes, e em cada face escreve a soma dos quatro nmeros das arestas adjacentes. E poss que a e u vel soma de todos os nmeros escritos no cubo seja 2004? u

Lista 6
Problema 26 Trs amigos planejam uma viagem ao campo de Rodrigo. Evaldo diz : Para chegarmos demoraremos no mximo 5 e a horas. Andra ento fala: Garanto que chegamos em mais de 5 horas. e a Raul diz: A viagem dura mais de 2 horas. Se apenas um deles tem razo, qual deles ? O que se pode dizer sobre a duraao da viagem? a e c Problema 27 Usando os algarismos 1, 2, 3, 4, 5, 6, 7, 8 e 9, cada um uma unica vez, forme 3 nmeros de trs algarismos de modo que u e o segundo nmero seja o dobro do primeiro e que o terceiro seja o triplo do primeiro. u Problema 28 (OBM/99) Na gura, os tringulos ABC e EGF so equilteros. O per a a a metro do tringulo ABC 132cm e, alm a e e disso, AE = EC, BD = DC, EF = F C e DG = GE. B

A (a) Qual o per e metro da rea sombreada? a

(b) Que fraao da rea do tringulo ABC representa a rea sombreada? c a a a

Problema 29 O retngulo da gura foi dividido em 9 retngulos menores, cujas reas esto indicadas: a a a a 4 5 Determine o valor da rea X. a Problema 30 Z guarda suas gurinhas em envelopes. Algumas gurinhas so quadradas e outras redondas. Em cada envelope e a ele pe apenas gurinhas do mesmo tipo. Os envelopes continham 9, 15, 16, 17, 26 e 34 gurinhas. Ontem Z deu o e um envelope de presente para seu irmo e agora o nmero de gurinhas redondas que tem o dobro do nmero de a u e u 2 3 1 X

28

gurinhas quadradas. Quantas gurinhas havia no envelope que ele deu? Qual o tipo de gurinha que h em cada a envelope restante?

Lista 7
Problema 31 Uma caixa contm 100 bolas, das quais 30 so vermelhas, 30 so azuis, 30 so verdes e das 10 restantes, algumas so e a a a a pretas e outras so brancas. Qual o nmero m a u nimo de bolas que devem ser retiradas da caixa, sem lhes ver a cor, para termos a certeza de que entre elas existem pelo menos 10 bolas da mesma cor? E para garantir que existem pelo menos 15 bolas da mesma cor? Problema 32 Quantos nmeros de 100 at 999 possuem todos os seus algarismos distintos e ordenados de maneira crescente? u e Problema 33 As diagonais AC e BD do quadriltero ABCD se encontram no ponto O. Os per a metros dos tringulos ABC e ABD a so iguais, assim como os per a metros dos tringulos ACD e BCD. Prove que AO = BO. a Problema 34 So constru a dos exteriormente ao Problema 35 Uma escola tem exatamente 100 armrios e 100 estudantes. No primeiro dia de aula os estudantes encontraram-se a fora do prdio e concordaram no seguinte plano: o primeiro estudante entrar na escola e abrir todos os armrios. O e a a a segundo aluno entrar e fechar todos os armrios com nmeros pares (2, 4, 6, 8...). O terceiro aluno, ento, inverter a a a u a a o que tiver sido feito a cada 3 armrios (3, 6, 9...), o que signica: ele abrir se o armrio estiver fechado ou fechar a a a a se o armrio estiver aberto. O quarto aluno inverter o que tiver sido feito a cada 4 armrios (4, 8, 12, ...) e assim a a a por diante. Aps todos os alunos terem entrado e realizado suas tarefas, quais armrios estaro abertos? (Olimp o a a ada Cearense/96) ABC, os tringulos equilteros ABM, BCN, ACP . Prove que N A = BP = CM . a a

Lista 8
Problema 36 (Cone Sul/92) Achar um nmero inteiro positivo n de dois algarismos de maneira que se colocarmos um 2 ` esquerda u a de n, e um 1 ` direita de n, o nmero resultante seja igual a 33n. a u Problema 37 Seja ABC um tringulo issceles com AB = AC e A = 36 . Traamos a bissetriz de B que corta AC em D. Traamos a o c c a bissetriz de BDC que corta BC em P . Marcamos agora um ponto R sobre a reta BC de modo que B seja o ponto mdio do segmento P R. Explique porque os segmentos RD e AP tm a mesma medida. e e Problema 38 Em um dado tringulo ABC, a altura AK, a mediana CM e a bissetriz BH se encontram em um ponto O, e alm a e disso AO = BO. Prove que o tringulo ABC equiltero. a e a Problema 39

29

(Rssia/2001) E poss u vel colocarmos nmeros inteiros positivos nas casas de um tabuleiro 9 2004 de modo que a u soma dos nmeros de cada linha e a soma dos nmeros de cada coluna sejam primos? Justique sua resposta. u u Problema 40 (Seletiva Rioplatense/2001) Em um torneio de futebol participaram os times A, B, C e D. Cada time enfrentou cada um dos outros trs exatamente um vez. A tabela abaixo mostra o nmero de pontos ganhos, o nmero de gols pr e e u u o o nmero de gols contra de cada equipe: u Clube A B C D PG 7 4 4 1 Gp 2 5 2 2 Gc 0 2 2 7

Cada equipe recebeu 3 pontos em caso de vitria, 1 ponto ao empatar e 0 ponto em caso de derrota. o Sabendo que o jogo C D teve pelo menos um gol, determine o placar do jogo B D. Considere todos os poss veis casos.

Lista 9
Problema 41 Ache o ultimo algarismo de N = 12 + 22 + 32 + 42 + ... + 992 . Problema 42 (Seletiva Rioplatense/04) E poss distribuirmos os nmeros 1, 2, 3, ..., 30 nas casas de um tabuleiro 5 6 de modo vel u que: (a) A soma dos nmeros de cada coluna seja igual? u (b) A soma dos nmeros de cada linha seja igual? u Problema 43 Um trapzio ABCD de bases BC e AD com BC < AD tal que 2.AB = CD e BAD + CDA = 120 . Determine e e os angulos do trapzio ABCD. e Problema 44 (Rioplatense/97) H 1997 nmeros escritos ao redor de uma circunferncia, dos quais 1996 so zeros e um deles 1. a u e a e A unica operaao permitida escolher um nmero e modicar seus dois vizinhos (de 0 para 1 e de 1 para 0). c e u (a) Demonstre que poss e vel, usando vrias vezes a operaao permitida, chegar a ter todos os nmeros iguais a 1. a c u (b) Decida se com 1998 nmeros, sendo 1997 zeros e um 1, poss chegar ao resultado da parte anterior. u e vel Problema 45 Fernando, Larissa e Gulherme esto formando uma roda e se divertindo com o seguinte jogo. Um deles escolhe um a nmero e o diz em voz alta, a pessoa que estiver ` sua esquerda o divide pelo seu maior divisor primo e diz o resultado u a em voz alta e assim sucessivamente. Ganha a pessoa que disser em voz alta o nmero 1, e o jogo termina. u Fernando escolheu um nmero maior que 50 e menor que 100 e ganhou. u Na partida seguinte, Guilherme escolheu o nmero seguinte ao escolhido por Fernando e tambm ganhou. u e Determinar todos os nmeros que possam ter sido escolhidos por Fernando. u

30

Lista 10
Problema 46 Onofre, tambm conhecido como Paladino, deseja cobrir um tabuleiro 7 7, e para isso dispe de dois tipos de peas e o c mostradas abaixo: (A) (B)

Onofre quer usar a menor quantidade poss vel de peas do tipo (B). Quantas peas do tipo (B) nosso heri vai c c o utilizar? D exemplo de uma cobertura usando o m e nimo poss de peas tipo (B). vel c Problema 47 3 DE 4 CE = e = . Sejam As diagonais AC e BD de um quadriltero convexo ABCD se cortam em E de forma que a AC 7 BD 9 P e Q pontos que dividem o segmento BE em trs partes iguais, com P entre B e Q, e R o ponto mdio do segmento e e AE. Calcule: area(AP QR) area(ABCD) Problema 48 (Rssia/98) Um nmero de quatro d u u gitos escrito no quadro-negro. As operaoes permitidas so: adicionar 1 a e c a dois d gitos vizinhos (caso nenhum deles seja 9), ou subtrair 1 de dois d gitos vizinhos (caso nenhum deles seja 0). E poss obtermos 2002 a partir de 1234 realizando algumas operaoes? vel c Problema 49 (Seletiva Rioplatense/04) Em cada casa de um tabuleiro 8 8 escrevemos um nmero inteiro. Sabe-se que para cada u casa, a soma dos seus vizinhos 1. Encontre a soma de todos os nmeros do tabuleiro. e u Nota: Consideramos vizinhas casas com um lado em comum. Problema 50 (Rioplatense/2003) Os 5 melhores times de futebol da Amrica participaram de um torneio, onde todos jogaram contra e todos uma unica vez. Da forma usual, em caso de vitria o time ganha 3 pontos, em caso de empate cada time ganha o 1 ponto e em caso de derrota o time ganha 0 ponto. Aps o nal do torneio foi confeccionada uma tabela especial para o a Olimp ada Rioplatense, que mostra o total de pontos de cada time (Pt), o nmero de vitrias (V), a quantidade de u o gols a favor (Gp) e a quantidade de gols contra (Gc), onde claro faltam alguns dados: e Clube Fortaleza River Plate Santos Boca Juniors Cruzeiro PG 10 8 5 3 ? V 3 2 1 1 ? Gp 4 7 2 1 0 Gc 0 1 ? 4 6

(a) Complete a tabela acima indicando o total de pontos ganhos e o total de vitrias do Cruzeiro, e o nmero de o u gols contra do Santos. (b) Construa toda a tabela de jogos do campeonato indicando, com justicativa, os placares de todas as partidas.

Cap tulo 4

Soluoes c
Soluo 1 ca Temos vrias soluoes para este problema. Aqui vo duas delas (comeando em um vrtice do tringulo e percorrendo a c a c e a os pontos no sentido anti-horrio): a 1, 6, 8, 2, 7, 5, 3, 4, 9 7, 3, 5, 8, 4, 2, 9, 6, 1 Obs: Seria interssante investigar mais de perto este problema e saber por exemplo quais valores pode assumir a soma S comum dos lados (no primeiro exmplo S = 17 e no segundo S = 23). Para isso veja as dicas oferecidas no problema do quadrado mgico (Problema 10). a Soluo 2 ca Digamos que as idades dos lhos sejam a, b, c. Temos portanto: a + b + c = 22 Daqui a x anos, as idades dos lhos sero a + x, b + x e c + x enquanto a idade do pai ser 40 + x. Queremos que: a a (a + x) + (b + x) + (c + x) = 40 + x (a + b + c) + 3x = 40 + x 22 + 2x = 40 x = 9

A resposta portanto 9 anos. e Soluo 3 ca Um ano no bissexto tem 365 dias. O dia do meio portanto seria o 183 . Basta ento contarmos: a a Janeiro : 31 dias Fevereiro: 28 dias Maro: 31 dias c Abril: 30 dias Maio : 31 dias Junho: 30 dias 31

32

At aqui temos 181 dias. Logo o 183 dia ser 02 de Julho. e a Soluo 4 ca Observe as duas ultimas armaoes: O caj est ` direia da caixa vermelha, que por sua vez est ` direita da banana. c a aa aa Logo devem estar dispostos assim: banana cx vermelha caj a Conclu mos portanto que o abacaxi deve estar na caixa vermelha, e que a caixa da banana a verde, pois na primeira e armaao ele diz que a caixa verde est ` esquerda da azul. A conguraao deve ser ento: c aa c a cx verde (banana) cx vermelha (abacaxi) cx azul (caj) a

Soluo 5 ca Neste problema voc poderia fazer uma anlise do seguinte tipo: e a Se Raul estiver falando a verdade, amanh ser realmente um dia de mentir. Nesse caso o dia da armaao a a c e uma sexta-feira. Se Raul estiver mentindo, ento amanh ser dia de falar a verdade. Nesse caso a armaao a a a c deve ter sido feita numa tera-feira. c Se Cida estiver falando a verdade, amanh ser realmente um dia de mentir, e o dia em que foi feita a frimaao a a c uma tera. Caso Cida esteja mentindo, amanh ser um dia de falar a verdade, e a armaao deve ter sido e c a a c feita num sbado. a Como os dois disseram a frase no mesmo dia, conclu mos que a resposta tera-feira. e c Soluo 6 ca Nosso heri pode usar a seguinte estratgia para obter os 4 litros exatos. Digamos que o balde A tenha capacidade o e para 5l, enquanto o balde B tem capacidade para 3l. 1. Encher o balde A 2. Derramar 3l no balde de B, sobrando 2l em A. 3. Seque ento o balde B e derrame os 2l restantes de A em B. Agora B cou com 2l e A cou vazio. a 4. Encha novamente o balde A e em seguida complete 1 litro que falta no balde B. Sobraro 4 litros exatos no a balde A. Obs: Voc pode experimentar outras variaoes deste problema, como por exemplo com baldes de 7l e 5l obter 4 e c litros... Soluo 7 ca E poss vel sim! Imagine que ele esteja falando a frase no dia 1 de janeiro e que seu aniversrio seja no dia 31 de a dezembro. Soluo 8 ca Para numerar as pginas de 1 at 99 ele vai usar 20 vezes o algarismo 2, dez vezes na casa das unidades: 2, 12, 22, ..., 92 a e e dez vezes na casa das dezenas: 20, 21, 22, ..., 29. Os prximos dois 2 s que vo aparecer sero nas pginas 102 e 112. o a a a Agora cuidado! Ele poder continuar numerando as pginas at a 119, que a resposta. a a e e Soluo 9 ca O enunciado diz que s h um cara que est falando a verdade. Fixemos as atenoes em Eduardo e Joo. Para o a a c a Eduardo temos duas possibilidades: ou ele mente, ou ele fala a verdade.

33

Se Eduardo mente, ento Joo estar falando a verdade. Como s pode haver um falando a verdade, esta pessoa a a a o s poder ser Eduardo ou Joo. De modo que temos ento certeza que Andr e Rafael mentiram. Como Rafael o a a a e mentiu, ele deve ser o culpado. Ento Eduardo deve estar mentindo tambm, e a pessoa que falou a verdade foi Joo. a e a Resposta: Rafael Soluo 10 ca O item (a) e o item (b) deste problema j foram resolvidos no cap a tulo 2. Vamos ento resolver o item (c). Vamos a inicialmente pr letras a, b, c, ..., p para representar os nmeros 1, 2, ..., 16 em alguma ordem: o u a e i m b f j n c g k o d h l p

Como a soma de cada linha deve ser igual, chamemos esta soma comum de S. Montando as equaoes teremos: c a+b+c+d e+f +g+h i+j+k+l m+n+o+p Somando estas quatro expresses obtemos: o a + b + c + ... + p 1 + 2 + ... + 16 136 S = 4S = 4S = 4S = 34 = S = S = S = S

H algumas maneiras diferentes de montar este tabuleiro, porm no vejo nenhum mtodo prtico para tal construao. a e a e a c Apresento a vocs trs soluoes descobertas por alunos do Projeto Numeratizar: e e c 4 13 16 1 6 11 10 7 9 8 5 12 15 2 3 14 10 2 13 9 1 15 8 7 12 6 5 4 11 16 3 14 1 10 7 16 14 8 9 3 15 5 12 2 4 11 6 13

Soluo 11 ca Observe que se considerarmos cada garrafa com 1 litro, haver 10, 5 litros a serem repartidos, o que d 3, 5 litros para a a cada garoto. Apresentamos aqui dois modos diferentes de se fazer a partilha: 1 modo: Garoto A: 3 cheias, 1 metade e 3 vazias. Garoto B: 3 cheias, 1 metade e 3 vazias. Garoto C: 1 cheia, 5 metades e 1 vazia. 2 modo: Garoto A: 2 cheias, 3 metades e 2 vazias.

34

Garoto B: 2 cheias, 3 metades e 2 vazias. Garoto C: 3 cheias, 1 metade e 3 vazias.

Soluo 12 ca Quando retiramos 1 litro de A para B, o barril B ca com 101 litros, numa mistura de 100 litros de lcool e 1 litro de a gua. Ao realizar o segundo passo, vamos tirar 1 litro da mistura de B, digamos que venha x de lcool e (1 x) de a a gua. Teremos ento ao nal: a a Barril A: (100 x) gua e x lcool. a a Barril B: (100 x) lcool e x gua. a a Concluso: H a mesma quantidade de lcool no barril A que de gua no barril B. a a a a Soluo 13 ca Queremos montar um nmero de 5 algarismos pal u ndromo: Usaremos o princ pio fundamental da contagem. Para a primeira posiao (mais ` esquerda) temos 9 possibilidades c a (exclui-se o ZERO). Uma vez escolhida esta posiao, temos 1 possibilidade para a ultima posiao (deve ser um d c c gito igual ao primeiro). Temos 10 escolhas para o d gito da segunda posiao. Uma vez escolhida esta segunda posiao, c c novamente temos 1 escolha para a quarta. E por ultimo temos 10 possibilidades para o d gito central. No total temos: 9 10 10 1 1 = 900 nmeros. u Soluo 14 ca Vamos reslver este problems de trs para a frente. a (d) Os cubinhos que tm trs faces pretas so exatamente os 8 cubinhos das esquinas. e e a (c) Os cubinhos que tm exatamente duas faces pretas esto numa aresta do cubo inicial, mas que no so ese a a a a quinas. Temos 8 cubinhos desse tipo em cada aresta, e 12 arestas, totalizando 96 destes cubinhos. (b)Os cubinhos que tm exatamente uma face preta esto nas faces do cubo inicial, mas no esto nas arestas. e a a a a Em cada face, temos 64 desse cubinhos. Como so 6 faces, temos 384 destes cubinhos. a (a) Os cubinhos com nenhuma face preta so os restantes: 1000 8 96 384 = 512. a Soluo 15 ca Observe que: O 1 o 1 termo. e O ultimo 2 o (1 + 2) termo. e O ultimo 3 o (1 + 2 + 3) termo. e Seguindo neste racioc nio, o ultimo nmero k o: u e 1 + 2 + 3 + ... + k = k(k + 1) 2

termo. Vamos procurar k de forma que isso chegue perto de 1999. Encontramos k = 62. O ultimo 62 ser o 1953 a termo da sequncia. Agora viro 63 termos iguais a 63. Em particular o 1999 termo ser um 63, e o resto da diviso e a a a deste cara por 5 3. e

35

Soluo 16 ca Faremos a contagem do seguinte modo: De 1 a 9: So 9 d a gitos usados. De 10 a 99: So 90 nmeros, cada um com 2 d a u gitos, ou seja 90 2 = 180 d gitos utilizados. De 100 a 999: So 900 nmeros, cada um com 3 d a u gitos, ou seja 900 3 = 2700 d gitos utilizados. Como estamos procurando pelo 2004 d gito escrito, este deve aparecer em um nmero de trs algarismos. Vamos u e calcular quantos d gitos escrevemos at o nmero de trs algarismos x. De 100 a x temos (x 100 + 1) 3 = 3x 297 e u e algarismo escritos, e no total de 1 at x temos: e 9 + 180 + (3x 297) = 3x 108 algarismos escritos. Fazendo: vemos que o 2004 algarismo escrito o 4 do nmero 704. e u Soluo 17 ca A unica coisa que precisamos para resolver este problema conhecer a representaao em base 10 dos nmeros!! Veja: e c u IRA + ARO + ORA = = = (100I + 10R + A) + (100A + 10R + O) + (100O + 10R + A) 102A + 101O + 100I + 30R ()

3x 108 = 2004 3x = 2112 x = 704

(a) Se queremos deixar esta soma maior poss vel, devemos substituir os maiores nmeros para as letras que tm maiores u e coecientes na expresso (). Ou seja A = 9; O = 8; I = 3; R = 1. A maior soma ser ento: a a a S = 102.9 + 101.8 + 100.3 + 30.1 = 2056 (b) Para o clculo da menor soma, o racioc a nio inverso. Devemos fazer A = 1; O = 3; I = 8; R = 9. A menor e soma portanto: e S = 102.1 + 101.3 + 100.8 + 30.9 = 1475

Soluo 18 ca (a) Quem possui a estratgia vencedora o segundo jogador, ou seja ele pode sempre vencer independente de como e e o primeiro jogue. Sua estratgia ser a seguinte: ele deve jogar sempre o oposto do primeiro. Assim se o 1 jogador e a move a cha 1 casa para a freente, o 2 deve mov-la 2 casas. E se o 1 move a cha 2 casas para a frente, o 2 deve e mov-la 1 casa apenas. e Dessa forma o 2 jogador vai completando o tabuleiro de 3 em 3 casas e ele (2 jogador) alcanar as casas mltiplos c a u de 3: 3, 6, 9, 12, 15, 18, 21, .... (b) No caso geral de um tabuleiro 1 N , se o N for mltiplo de 3, o 2 jogador vence com a estratgia apontada acima. u e Caso N no seja mltiplo de 3, quem tem a estratgia para vencer o 1 jogador. Na sua primeira jogada ele deve a u e e deixar a quantidade de casas restantes mltiplo de 3 (ou seja se o resto de N por 3 for 1, ele joga 1 casa, e se o resto u de N por 3 for 2 ele joga 2 casas). Em seguida o 1 jogador s deve usar a estratgia acima de sempre completar 3 casas. o e Obs: Voc pode experimentar variaoes deste jogo, onde as jogadas permitidas so: mover a cha 1, 2, 3, ..., ou k e c a casas para a frente. Nesse caso a estratgia vencedora tem a ver com a diviso por (k + 1) e a dica sempre completar e a e (k + 1) casas aps o movimento do adversrio. o a Soluo 19 ca

36

Digamos que haja x pessoas famintas e y pessoas normais. Como 4 pizzas no so sucientes, nem que cada pessoa a a coma o seu m nimo de pedaos, dar certo. Ento devemos ter: c a a 6x + 2y > 48 Por outro lado, com 5 pizzas sobrariam pedaos. Isso quer dizer que mesmo que cada pessoa coma seu mximo de c a pedaos, o total de 60 pedaos ser atingido, ou seja: c c a 7x + 3y < 60 Vamos agora testar os poss veis casos. Da segunda equaao obtemos que x < 9. Da c : x = 8, devemos ter y = 1 que funciona!!!! x = 7, pela primeira equaao y 4, o que no funcionaria na segunda. c a x = 6, pela primeira equaao, obtemos y 7, o que no funcionaria na segunda c a os casos x = 5, 4, 3, 2, 1 tambm no funcionaro. e a a Conclu mos portanto que 9 pessoas foram ` pizzaria, das quais 8 estavam famintas. a Soluo 20 ca Observe que ao tomarmos um tringulo equiltero e ligarmos seus pontos mdios estamos dividindo este em 4 tringulos a a e a equilteros menores. Se o tringulo equiltero original j faz parte de uma diviso, estaremos aumentando a quantia a a a a dade de tringulos da diviso em 3. a a (a) Usando a dica acima, ao construirmos uma diviso do tringulo original em 1 tringulo (ele mesmo!), conseguiremos a a a dividi-lo em 22 triangulos menores, j que 22 = 3.7 + 1. Veja a gura abaixo: a

(b) Para construirmos uma diviso em 17 triangulinhos, primeiro conseguimos dividir em 8 e depois acrescentamos a mais 9 com a dica acima. A gura nal pode car assim:

Soluo 21 ca Desenvolvendo temos: 1094 94 = 1000...00 94 = 999..99 06


94 zeros 92 noves

37

A soma dos algarismo portanto: e (92 9) + 6 = 834

Soluo 22 ca Veja que 1722 = 2 3 7 41. As unicas possibilidades de multiplicarmos dois nmeros de dois algarismos para obter u 1722 so: a 1722 = 42 41 = 82 21 Vamos agora ver quais so os verdadeiros resultados destas multiplicaoes: a c 42 41: O verdadeiro resultado seria 24 21 = 504. 82 21: O verdadeiro resultado seria 64 41 = 2624. Como dito no enunciado que o verdadeiro resultado seria um nmero de trs algarismos, conclu e u e mos que Onofre queria multiplicar os nmeros 24 e 21. u Soluo 23 ca Veja a gura abaixo: Q z x A

z B

P y D y

x C

Como o ABC issceles, sabemos que AD tambm mediatriz, logo P B = P C = P Q. Veja os angulos marcados e o e e na gura. No BQC temos que: 2x + 2y + 2z = 180 x + y + z = 90 E da conclu mos que: P QC = x = 90 (y + z) = 90 75 = 15

Soluo 24 ca Fatorando em primos cada nmero do conjunto {1, 2, ..., 28} e multiplicando todos esses nmeros, encontramos: u u P = 225 .313 .56 .74 .112 .132 .17.19.23 Para que o produto seja um quadrado perfeito todos os expoentes devem ser pares. Devemos portanto tirar os nmeros 17, 19, 23 e ainda um fator 2 e um fator 3 que pode ser feito de uma vez s tirando o nmero 6. A reposta u o u e portanto 4 nmeros. u Soluo 25 ca Sejam a, b, c, d, e, f, g, h os oito nmeros escritos nos vrtices do cubo. Seja S a soma total dos nmeros dos vrtices, u e u e das arestas e das faces do cubo. Nas 3 arestas incidentes no vrtice a aparece uma parcela a em cada uma. Nas trs e e faces incidentes no vrtice a aparece uma parcela 2a em cada uma. O mesmo vale para os outros nmeros, de modo e u que: S = 10(a + b + c + d + e + f + g + h)

38

Faa uma gura bem direitinho para vericar isto!! Veja que a soma total mltipla de 10, e portanto no pode ser c e u a igual a 2004. Soluo 26 ca Temos trs possibilidades a analisar: e Se viagem dura no mximo 2 horas: Nesse caso apenas Evaldo teria razo. a a Se a viagem dura mais que 2 horas e no mximo 5 horas: Nesse caso Evaldo e Raul teriam razo. a a Se a viagem dura mais que 5 horas: Nesse caso Andra e Raul teriam razo. e a Como dito no enunciado que apenas um deles tem razo, conclu e a mos que a primeira das trs possibilidades acima e e a verdadeira. Soluo 27 ca Duas possibilidades so: a 327, 654 e 981. 273, 546 e 819.

Soluo 28 ca Observando a gura do enunciado vemos que: B

A E F C (a) O lado do ABC mede 44. Veja agora que AE = 22, EF = 11, F G = 11, GD = 11, BD = 22. O per metro da rea sombreada portanto: a e BA + AE + EF + F G + GD + DB = 44 + 22 + 11 + 11 + 11 + 22 = 121cm 3 1 (b) A rea ABDE vale da rea do ABC. J a rea do EGF vale da rea do a a a a a 4 4 da rea do ABC. Logo a razo entre a rea sombreada e a rea do ABC : a a a a e 3 1 1 3 1 13 + . = + = 4 4 4 4 16 16 EDC que por sua vez vale 1 4

Soluo 29 ca Se dois retngulos tm mesma base, a razo entre suas reas a mesma rezo entre suas alturas. Do mesmo modo, se a e a a e a dois retngulos tm mesma altura, a razo entre suas reas a mesma razo entre suas bases. Veja gura abaixo: a e a a e a

39

b 2

a 1 d e X f

4 5

O retngulo de rea 3 tem mesma base que o retngulo de rea 2 (base b), assim: a a a a 3 3d e = e= d 2 2 Veja agora que o retngulo de rea 5 tem mesma base que o retngulo de rea 4 (base c), e com isso: a a a a 5 5e 15d f = f = = e 4 4 8 Lembrando que ad = 1, a rea procurada vale: a X = af = 15 15 ad = 8 8

Soluo 30 ca Compreendendo bem o enunciado do problema, devemos retirar um pacote para que os pacotes restantes possam ser divididos em dois grupos um com o dobro de gurinhas do outro. Digamos que o pacote retirado tenha x gurinhas e que os pacotes restantes foram divididos em dois grupos: um com y gurinhas e outro com 2y gurinhas. O total de gurinhas inicialmente : e S = 9 + 15 + 16 + 17 + 26 + 34 = 117 Quando tiramos x devemos ter: 117 x = y + 2y = 3y Assim, x deve ser mltiplo de 3. Temos duas possibilidades: u Se x = 9: Da pela equaao () conclu , c mos que y = 36. Porm no h modo de formarmos um grupo de 36 e a a gurinhas com pacotes de 15, 16, 17, 26 e 34. Se x = 15: Pela equaao () descobrimos que y = 34. Agora sim! O unico modo de formarmos um grupo com 34 c gurinhas dentre as restantes considerar o pacote de 34 como um grupo s, e os pacotes de 9, 16, 17, 26 como e o o outro grupo. Concluso: O pacote que Z deu ao seu irmo tinha 15 gurinhas. Dos pacotes restantes o de 34 de gurinhas a e a e quadradas, enquanto os pacotes de 9, 16, 17, 26 so de gurinhas redondas. a Soluo 31 ca (a) Se tirarmos 37 bolas, pode ser que venham 9 vermelhas, 9 azuis, 9 verdes, e 10 pretas e brancas (nem todas pretas, nem todas brancas) de modo que no ter a amos 10 bolas da mesma cor. Agora se tirarmos 38 bolas, nem que venham as 10 pretas/brancas, teremos pelo menos 28 dentre vermelhas, azuis e verdes, o que j garante a existncia de 10 da a e mesma cor. (b) Seguindo o mesmo racioc nio acima, devemos tirar 53 bolas para garantir que h 15 de uma mesma cor. a Soluo 32 ca Primeiro vamos contar os nmeros bons que comeam com 1: u c ()

40

Comeam por 12: 7 nmeros. c u Comeam por 13: 6 nmeros. c u Comeam por 14: 5 nmeros. c u Comeam por 15: 4 nmeros. c u Comeam por 16: 3 nmeros. c u Comeam por 17: 2 nmeros. c u Comeam por 18: 1 nmeros. c u De modo que comeando com 1 temos: c 7+6+5+4+3+2+1= Seguindo o mesmo racioc nio temos: Comeando por 2: 6 + 5 + 4 + 3 + 2 + 1 = 21 nmeros. c u Comeando por 3: 5 + 4 + 3 + 2 + 1 = 15 nmeros. c u Comeando por 4: 4 + 3 + 2 + 1 = 10 nmeros. c u Comeando por 5: 3 + 2 + 1 = 6 nmeros. c u Comeando por 6: 2 + 1 = 3 nmeros. c u Comeando por 7: 1 nmero. c u No total temos portanto: 28 + 21 + 15 + 10 + 6 + 3 + 1 = 84 nmeros u 7.8 = 28 2

Soluo 33 ca Observe a gura abaixo: A B

D Como os per metros dos tringulos ABC e ABD so iguais temos: a a AB + BC + AC AC + BC AD + CD + AC AD + AC Somando () + () temos:

= AB + AD + BD = AD + BD ()

Agora, como os per metros dos tringulos ACD e BCD tambm so iguais: a e a = BC + CD + BD = BC + BD ()

AC + BC + AD + AC = AD + BD + BC + BD AC = BD

41

Com isso, olhando para () conclu mos tambm que: e BC = AD Por m, perceba que os tringulos ABD e ABC so congruentes pelo caso LLL e com isso os angulos CAB e DBA a a so iguais, o que torna o tringulo AOB issceles e portanto AO = BO. a a o Soluo 34 ca Atenao para a gurinha: c P A M B 60 C

60

N O que voc deve perceber acima a congruncia entre os tringulos ACN e BCP , pelo caso LAL: e e e a AC ACN CN = CP = BCP = 60 + = BC

Com isso provamos que os lados opostos ao angulo de (60 + ) nos dois tringulos so iguais, ou seja AN = BP . De a a modo anlogo podemos provar que AN = CM , e com isso: a AN = BP = CM

Soluo 35 ca Observe que ao nal do processo, cada armrio d foi movimentado uma quantidade N (d) vezes, onde N (d) o nmero a e u de divisores positivos de d. Os armrios que caram abertos so aqueles que foram operados uma quantidade a a mpar de vezes, ou seja, aqueles nmeros que possuem uma quantidade u mpar de divisores. E bem conhecidada a frmula o para calcular a quantidade de divisores positivos de um nmero fatorado em primos d = p 1 p2 ...pk : u 1 2 k N (d) = (1 + 1)(2 + 1)...(k + 1) Para que N (d) seja mpar, necessrio portanto que cada i seja par. Isso faz de d um quadrado perfeito. Logo os e a armrios buscados so os de nmero: a a u 1, 4, 9, 16, 25, 36, 49, 64, 81, 100

Soluo 36 ca

42

e Digamos que o nosso nmero seja n = ab = 10a + b. O que dito no enunciado traduz-se para: u 2ab1 = 33 ab

2000 + 100a + 10b + 1 = 33(10a + b) 2001 = 230a + 23b = 23(10a + b) 87 = 10a + b = n A resposta portanto n = 87. e Soluo 37 ca Comeamos desenhando uma boa gurinha: c A
36

D
36 36 36 36

72 72

O tringulo ABC issceles, logo CBA = BCA = 72 . Como BD bissetriz temos que DBA = DBC = 36 . a e o e Isso faz com que o tringulo ABD seja issceles e AD = BD (marcados na gura com duas bolinhas). a o Alm disso, observe que os angulos BDP = P DC = 36 . Com isso o tringulo BDP tambm issceles e e a e e o temos BP = DP . Por construao RB = BP (marcamos estes segmentos com uma bolinha na gura). Observe ento c a a congruncia dos tringulos ADP e DBR pelo caso LAL: e a AD = BD ADP = DBR = 144 DP = BR Com isso, os lados opostos aos angulos de 144 nesses tringulos so iguais, ou seja AP = RD. a a Soluo 38 ca Observe a gura: A

H M O

B Sabemos que o ABO issceles, logo: e o

K ABO = BAO =

Como BH bissetriz, conclu e mos que OBC = . Olhando ento para a soma dos angulos no a 3 = 90 = 30

ABK chegamos a:

43

Alm disso, no ABO, OM mediana e portanto altura tambm, j que este tringulo issceles. Da CM A = 90 e e e e a a e o e CM passa a ser mediana e altura no tringulo ABC, o que torna este tringulo tambm issceles, com CAB = a a e o ABC = 60 . O ABC deve ser portanto equiltero. a Soluo 39 ca Suponha que seja poss fazer tal construao. Sejam L1 , L2 , ..., L9 as somas dos nmeros de cada uma das 9 linhas, vel c u e C1 , C2 , C3 , ..., C2004 as somas dos nmeros de cada uma das 2004 colunas. u Como cada Li e Cj so primos, estes devem ser nmeros a u mpares (j que so soma de pelo menos nove inteiros a a positivos). Seja S a soma de todos os nmeros do tabuleiro. Por um lado ter u amos: S = L1 + L2 + ... + L9 donde conclu mos que S e mpar, pois soma de 9 e mpares. Por outro lado: S = C1 + C2 + ... + C2004 e daqui concluir amos que S par, o que um absurdo. e e Logo tal construao no poss c a e vel. Soluo 40 ca Inicialmente observe que cada time jogou trs vezes. Reproduzimos aqui a tabela do campeonato: e Clube A B C D PG 7 4 4 1 Gp 2 5 2 2 Gc 0 2 2 7

Observando as pontuaoes acima podemos concluir que: c A obteve 2 vitrias e 1 empate. o B obteve 1 vitria, 1 empate e 1 derrota. o C obteve 1 vitria, 1 empate e 1 derrota. o D obteve 1 empate e 2 derrotas. Como o time A s fez 2 gols, suas duas vitrias devem ter sido por 1 0, e seu empate por 0 0. Temos trs casos a o o e considerar, dependendo contra quem foi o empate de A: 1 caso: A empata com B Nesse caso j sabemos que: a A A A 00 10 10 B C D

O outro empate do campeonato deve ter sido C D. Assim C deve ter vencido B, enquanto B vence D. Como dito e no enunciado, o empate C D com gols. Pode ter sido 1 1 ou 2 2 j que C s faz dois gols. Porm, se tivesse e a o e sido 2 2, C j levaria 3 gols (pois levou 1 de A) o que no pode. Logo, chegamos a mais um resultado: a a C 11 D

44

Com isso sobra 1 gol apenas para C fazer em sua vitria, donde conclu o mos que: B 01 C E contando os gols que faltam, chegamos a: B 51 D 2 caso: A empata com C Nessa opao sabemos que: c A A A 10 00 10 B C D

Nesse caso B D deve ser empate, como B precisa de 1 vitria, B deve ter vencido C, e com isso C deve ter vencido o D. Agora observe que B deve marcar 5 gols. Para vencer C, B no pode marcar mais de 2 gols, pois C s leva 2 gols. a o Assim B deve fazer pelo menos 3 gols em D, mas a esse jogo no poderia ser empate j que D s marca 2 gols. Este a a o caso no tem soluao. a c 3 caso: A empata com D Agora teremos a seguinte situaao: c A A A 10 10 00 B C D

O outro empate deve ser ento entre B e C. Como D j empatou com A, D deve perder para B e C. Como D no a a a leva gol de A, deve levar seus 7 gols de B e C. Porm B e C marcam juntos exatamente 7 gols, que devem ento ser e a disparados todos em D. Achamos portanto o resultado do empate B C: B 00 C Como B deve levar 2 gols, ser 1 do A e 1 do D, e portanto temos os placares que faltam: a B 51 D C 21 D Curiosamente, a resposta a mesma para os dois poss e veis casos: B 5 1 D. Soluo 41 ca Denotaremos por ..x um nmero que acaba no algarismo x. Veja que a sequncia dos ultimos algarismos se repete de u e dez em dez: N N N (..1 + ..4 + ..9 + ..6 + ..5 + ..6 + ..9 + ..4 + ..1 + ..0) + (..1 + ..4 + ... 10(..1 + ..4 + ..9 + ..6 + ..5 + ..6 + ..9 + ..4 + ..1 + ..0) 10.(..5) 0

Vemos portanto que N acaba em ZERO. Soluo Alternativa: Usando a frmula para o clculo da soma dos quadrados temos: ca o a N= 99.100.199 33.50.199 0 (mod 10) 6

45

Soluo 42 ca Veja que a soma de todos os nmeros do tabuleiro : u e S = 1 + 2 + ... + 30 = 30.31 = 15.31 = 465 2

(a) Como 465 no divis por 6 (quantidade de colunas) no poss distribuir os nmeros no tabuleiro para a e vel a e vel u que a soma de cada coluna seja igual. (b) Em contrapartida, como 465 = 93 5, j temos um ind a cio de que poss e vel fazer a distribuiao para deixar c a soma de cada uma das 5 linhas igual (a 93). O exemplo abaixo mostra que de fato isto poss e vel: 1 4 7 10 13 30 27 24 21 18 2 5 8 11 14 29 26 23 20 17 3 6 9 12 15 28 25 22 19 16

Soluo 43 ca Observe a gura abaixo: B C


60

x M x

x A

Por C traamos uma paralela ao lado AB encontrando AD em J. Veja que: c CJD + CDJ = BAD + CDA = 120 JCD = 60 Seja M o ponto mdio de CD. O JCM issceles, pois JC = CM = x, e tem angulo do vrtice C = 60 . Com e e o e isso o JCM deve ser equiltero e teremos JM = x. Observe que: a JM = CM = DM = x Conclu mos portanto que o CJD deve ser retngulo em J, o que faz com que = 90 e = 30 . Os outros angulos a dos trapzio so BCD = 150 e CBA = 90 . e a Soluo 44 ca (a) Considere um bloco de 4 zeros consecutivos na circunferncia: e 0000 Se realizarmos a operaao no segundo zero do bloco, modicaremos seus dois vizinhos e caremos com: c 1010 Agora se realizarmos a operaao na terceira posiao (o 1) modicaremos seus dois vizinhos para car com: c c 1111 Conseguimos ento com estes dois passos trocar quatro zeros consecutivos por quatro 1 s. Basta ento dividir os 1996 a a zeros em 499 blocos de 4 zeros consecutivos e aplicar o esquema acima. (b) Com 1997 zeros e um 1 inicialmente, no poss transformar tudo em 1. Para provar isso considere a soma S a e vel dos nmeros da circunferncia. Vejamos o que pode acontecer com S quando realizamos uma operaao: u e c

46

Se a casa operada tem dois vizinhos 0: Nesse caso S se transformar em S + 2. a Se a casa operada tem um vizinho 0 e outro vizinho 1: A soma S permanecer constante. a Se a casa operada tiver dois vizinhos 1: A soma S diminuir para S 2. a Veja que quando realizamos a operaao, a paridade de S no muda. Inicialmente S = 1. Logo, no podemos ter ao c a a nal todos os nmeros iguais a 1, pois nesse caso ter u amos S = 1998, que seria uma contradiao. c Soluo 45 ca Observe que o jogador que diz o nmero inicial N ganha o jogo quando a soma dos expoentes da fatoraao em primos u c de N mltipla de 3 (pois a cada rodada um fator primo sai, como se um expoente baixasse 1). A maneira mais na e u e raa de se fazer este problema ento fatorar todos os nmeros entre 50 e 100 e ver os pares de nmeros consecutivos c e a e u u cujas somas dos expoentes sejam mltiplas de 3. Aps esta anlise de casos, encontramos as respostas (o primeiro u o a nmero o que foi dito por Fernando): u e 63 = 32 .7 75 = 3.52 98 = 2.72 e e e 64 = 26 76 = 22 .19 99 = 32 .11

Soluo 46 ca Digamos que Onofre v utilizar x peas do tipo (A) e y peas do tipo (B) para cobrir o tabuleiro. Cada pea do tipo a c c c (A) tem 4 casas enquanto cada pea do tipo (B) tem 3 casas. O tabuleiro 7 7 tem 49 casas, o que nos permite c montar a seguinte equaao: c 4x + 3y = 49 Como y deve ser o menor poss vel, testemos estes valores: y = 0 4x = 49, y = 1 4x = 46, y = 2 4x = 43, Absurdo! Absurdo! Absurdo!

y = 3 4x = 40 x = 10 Pelo que foi visto acima, no podemos cobrir o tabuleiro usando apenas 0, 1 ou 2 peas (B). O menor valor poss a c vel seria 3, como mostra o exemplo abaixo:

Soluo 47 ca Para resolver este problema usaremos o seguinte resultado: se dois tringulos tm mesma altura, a razo entre suas a e a reas a mesma razo entre suas bases. Em particular quando um tringulo dividido em dois por meio de uma a e a a e ceviana, a razo entre suas reas a mesma entre suas bases. Veja a gura abaixo: a a e

47

C D E Q R P B 4 DE 4 DE = , temos que = . Assim: BD 9 BE 5

A Vamos denotar reas por parnteses. Chamemos (AEB) = x. Como a e DE 4 (ADE) = = BE 5 (AEB) donde conclu mos que (ADE) = 4x . Pelo mesmo racioc nio, temos: 5

(CEB) CE 3 3x = = (CEB) = (AEB) AE 4 4 Ainda de modo anlogo: a DE 4 4 3x 3x (CED) = = (CED) = . = (CEB) BE 5 5 4 5 Logo a rea do quadriltero ABCD ser: a a a (ABCD) = = = (ABE) + (ADE) + (CEB) + (CED) x+ 63x 20 EQA e 4x 3x 3x + + 5 4 5

Uma mediana divide um tringulo em duas partes de mesma rea. Sabendo disso, como QR mediana no a a e AP mediana no QAB temos: e (ERQ) = (RQA) e (QAP ) = (P AB) e isso claramente implica em: (AP RQ) = A resposta para o nosso problema portanto: e (AP QR) x 20 10 = . = (ABCD) 2 63x 63 1 x (ABE) = 2 2

Soluo 48 ca O que ser que no muda em um nmero quando adicionamos ou subtra a a u mos 1 unidade de dois d gitos vizinhos? Por meio desta operaao veja que podemos estar somando ou subtraindo: 11, 110 ou 1100 do nmero original. E o c u que estas parcelas tm todas em comum? So mltiplas de 11! Ou seja estamos sempre adicionando ou subtraindo e a u um nmero mltiplo de 11 do nmero corrente, de modo que o resto da diviso por 11 do n mero escrito no u u u a u quadro no muda. a Como comeamos com 1234 que deixa resto 2 por 11, no podemos chegar a 2002 que deixa resto 0 por 11. c a

48

Soluo 49 ca Observe as casas marcadas no tabuleiro abaixo: * * * * * * * * * * * * * * * * * * * *

Se olharmos para os vizinhos das casas marcadas acima, vemos que eles cobrem todo o tabuleiro e de maneira disjunta! Como a soma dos vizinhos de cada casa 1, a soma total dos nmeros do tabuleiro ser igual ao nmero de casas e u a u marcadas, que 20. e Soluo 50 ca Reveja a tabela do campeonato: Clube Fortaleza River Plate Santos Boca Juniors Cruzeiro PG 10 8 5 3 ? V 3 2 1 1 ? Gp 4 7 2 1 0 Gc 0 1 ? 4 6

(a) Inicialmente veja que o total de gols marcados por todos os times deve ser igual ao total de gols sofridos por todos os times: Gp = 4 + 7 + 2 + 1 + 0 = 0 + 1+? + 4 + 6 = Gc Conclu mos portanto que o Santos sofreu 3 gols. O campeonato teve ao todo 10 jogos, como indicado na tabela, 7 deles j acabaram com vitrias (3 + 2 + 1 + 1). e a o Veja ainda que Fortaleza teve 1 empate, River teve 2 empates e Santos teve 2 empates, o que somados j fornecem a 5 empates, ou seja pelo menos 3 jogos acabaram empatados. Esses so os 3 jogos restantes. Logo o Cruzeiro teve 0 a vitrias. o Vejamos agora o total de pontos do campeonato, houve 7 jogos com vitria (3 pontos cada) e 3 jogos empatados (2 o pontos cada) o que d um total de: a (7 3) + (3 2) = 27 pontos Portanto o Cruzeiro deve ter feito 1 ponto. (b) Vamos agora montar a tabela de jogos do campeonato. Fortaleza teve 3 vitrias e 1 empate. Esse empate o deve ter sido com o River que tambm no perdeu (2 vitrias e 2 empates). Como o Fortaleza no sofreu gols, tiramos: e a o a F ortaleza 0 x 0 River Logo o Fortaleza venceu Santos, Boca e Cruzeiro. O Boca obteve apenas uma vitria que deve ter sido sobre o Cruzeiro o (pois o jogo Boca x Cruzeiro no admite outro resultado). Sendo assim River e Santos vencem o Boca. Para o Santos a restam 2 empates que so com River e Cruzeiro. O ultimo jogo que River e Cruzeiro acaba com vitria do River. a e o Vamos agora aos placares. O empate Cruzeiro x Santos foi sem gols, pois o Cruzeiro no marcou gols no campeonato: a Cruzeiro 0 x 0 Santos Como o Boca marca s um gol no campeonato, este deve ser para consolidar sua vitria, da o o : Boca 1 x 0 Cruzeiro

49

O Fortaleza tem 4 gols para fazer 3 vitrias, logo duas delas sero 1 x 0 e outra ser 2 x 0. Se a vitria contra o Santos o a a o fosse de 1 x 0, ao Santos sobrariam 2 gols pro e 2 gols contra para uma vitria e um empate o que no poss o a e vel. Logo: F ortaleza 2 x 0 Santos E ainda: F ortaleza 1 x 0 Boca F ortaleza 1 x 0 Cruzeiro Resta ao Santos sofrer 1 gol, que deve ser para o River, j que o Boca no marca mais. Como o jogo Santos x River a a termina empatado devemos ter: Santos 1 x 1 River A vitria do Santos ser ento de: o a a Santos 1 x 0 Boca Resta ao Boca levar 2 gols do River, e no fazer nenehum, logo: a River 2 x 0 Boca Por m, o River deu uma goleada no Cruzeiro, para completar seus gols pro: River 4 x 0 Cruzeiro

50

Apndice A e

Sugestes de Conte do o u
Aqui vo ento algumas sugestes para que voc monte seu programa de ensino: a a o e

4a srie e
Carga horria proposta: 3h/aula por semana - curso de 40 semanas a 1. Teoria dos conjuntos - 9h 2. Nmeros inteiros (operaoes); Nmeros fracionrios; Expresses Numricas - 12h u c u a o e 3. Divisores de um nmero; Nmeros primos; Fatoraao em primos; Mtodo para encontrar divisores; Regras de u u c e divisibilidade; MDC; Algoritmo de Euclides; MMC - 24h 4. Fraoes II (mais problemas); Bases de Numeraao; D c c zimas Peridicas; Proporoes, Regra de trs - 24h o c e 5. Equaoes, manipulaoes algbricas com uma varivel; Sistemas de equaoes - 12h c c e a c 6. Geometria plana: Ponto, reta, segmentos de reta; Sistema mtrico decimal; Areas de guras planas e problemas; e Volumes de slidos, problemas - 21h o 7. Aulas de problemas; Simulados - 18h

5a srie e
Carga horria proposta: 4h/aula por semana - curso de 40 semanas a Aritmtica: e 1. Teoria dos Conjuntos - 8h 2. Nmeros inteiros e fracionrios, expresses numricas; Bases de Numeraao e d u a o e c zimas peridicas - 16h o 3. Proporoes e Regra de trs - 10h c e 4. Divisores de um nmero, Nmeros primos, Fatoraao em primos, Mtodo para encontrar divisores, Regras de u u c e divisibilidade, MDC, Algoritmo de Euclides, MMC - 22h 5. Equaoes com 1 varivel e Sistemas de equaoes - 12h c a c Geometria: 1. Ponto, reta, segmento de reta, angulos; Sistema mtrico decimal; Areas de guras planas e problemas; Volumes e de slidos, problemas - 28h o

51

52

2. Paralelismo e angulos - 12h 3. Congruncia de tringulos - 12h e a 4. Teorema de Pitgoras - 4h a Combinatria: o 1. Introduao aos princ c pios da contagem - 12h 2. Princ pio da casa e dos pombos, Lgica Matemtica; Paridade, Jogos e Invariantes - 24h o a

6a srie e
Carga horria proposta: 4h/aula por semana - curso de 40 semanas a Aritmtica e 1. Teoria dos Conjuntos - 6h 2. Nmeros inteiros e fracionrios, expresses numricas; Bases de Numeraao e d u a o e c zimas peridicas - 12h o 3. Proporoes e Regra de trs - 6h c e 4. Equaoes com 1 varivel e Sistemas de equaoes - 6h c a c 5. Divisores de um nmero, Nmeros primos, Fatoraao em primos, Mtodo para encontrar divisores, Regras de u u c e divisibilidade, MDC, Algoritmo de Euclides, MMC - 16h 6. Algoritmo da Diviso - 8h a Geometria(Bibliograa: Matemtica Elementar, vol 9) a 1. Introduao, segmento de reta; Angulos; Reviso: Sistema Mtrico, Areas, Volumes - 12h c a e 2. Tringulos, Congruncia de tringulos, Desigualdades no tringulo - 12h a e a a 3. Paralelismo; Perpendicularidade - 12h 4. Quadrilteros Notveis (8h); Circunferncia, rea e per a a e a metro (4h); Pontos Notveis no tringulo (8h) - 20h a a 5. Semelhana de tringulos e Teorema de Pitgoras - 10h c a a Combinatria-Algebra o 1. Introduao aos problemas de Olimp c ada - 6h 2. Princ pios da Contagem, Permutaoes, Combinaoes, Permutaoes Circulares - 14h c c c 3. Nooes de Algebra: Produtos Notveis, Ra c a zes e Potncias, Mdulos, Racionalizaao de denominadores - 10h e o c 4. Princ pio da casa e dos pombos; Paridade; Jogos e Invariantes - 10h

7a srie e
Carga horria proposta: 4h/aula por semana - curso de 40 semanas a Algebra 1. Introduao ` Lgica Matemtica - 4h c a o a

53

2. Produtos Notveis e fatoraoes - 8h a c 3. Radiciaao, Potenciaao, Funao Modular; Reviso: Equaoes do 1o grau, Sistemas de equaoes, Proporoes, c c c a c c c Regras de trs, muitos e muitos problemas - 10h e 4. Reviso: Bases de Numeraao, D a c zimas peridicas - 6h o 5. Equaoes do 2o grau (tudo) e funao quadrtica - 10h c c a 6. Progresses Aritmticas e Geomtricas - 6h o e e 7. Desigualdades entre as mdias - (se der tempo) e Geometria (Bibliograa: Matemtica Elementar, vol.9) a 1. Reviso: Introduao, retas, segmentos de reta, angulos; Reviso: Sistema mtrico decimal, per a c a e metros, reas e a volumes - 8h 2. Tringulos: Congruncias e Desigualdades (8h); Paralelismo e Perpendicularidade (6h); Quadrilteros Notveis a e a a (4h); Pontos notveis no tringulo (6h) - 24h a a 3. Pol gonos: Diagonais e angulos (2h); Circunferncia: Posioes, angulos, tangentes, quadrilteros inscrit e c a veis (8h) - 10h 4. Teorema de Tales e Teorema da bissetriz (4h); Semelhana de tringulos, potncia de ponto (6h); Tringulo c a e a retngulo: Relaoes Mtricas e Teorema de Pitgoras (6h) - 16h a c e a Aritmtica - Combinatria e o 1. Reviso: Nmeros inteiros, fracionrios, radicais, potncias, mdulos, d a u a e o zimas peridicas - 4h o 2. Divisores de um nmero, Nmeros primos, Fatoraao em primos, Mtodo para encontrar divisores, Regras de u u c e divisibilidade, MDC, Algoritmo de Euclides, MMC - 10h 3. Algoritmo da Diviso e Congruncias (introdutrio) (12h); Equaoes diofantinas (4h) - 16h a e o c 4. Princ pios da Contagem, Permutaoes, Combinaoes e Permutaoes Circulares - 12h c c c 5. Princ pio da Induao - 6h c 6. Paridade, Jogos e Invariantes, Princ pio da Casa e dos Pombos - 10h

8a srie e
Carga horria proposta: 6h/aula por semana - curso de 40 semanas a Geometria Bibliograa: Matemtica Elementar, vol 9; a Challenging Problems in Geometry, I.M. Yaglom Geometry Revisited, H.S.Coxeter Programa: 1. Tringulos: Congruncias e Desigualdades; Paralelismo e perpendicularidade - 16h a e 2. Quadrilteros Notveis (4h); Pontos notveis no tringulo (propriedades bsicas); Teorema de Tales, Teorema a a a a a da bissetriz, semelhana de tringulos (8h); Tringulos Retngulos e o teorema de Pitgoras (4h) - 16h c a a a a

54

3. Circunferncia: angulos, tangentes, potncia de ponto; Quadrilteros inscrit e e a veis e o teorema de Ptolomeu - 14h 4. Teoremas de Ceva e Menelaus - 6h 5. Relaoes mtricas no tringulo qualquer; Areas; Lei dos senos e Trigonometria - 14h c e a 6. Transformaoes Geomtricas: Homotetia, Rotaao, Reexo, Translaao - (opcional); O tringulo e seus c c e c a c a rculos: tringulo rtico, tringulo medial, o inc a o a rculo e exinc rculos, tringulo pedal, reta de Simpson, a reta de Euler e a o c rculo dos nove pontos - (opcional) Algebra 1. Produtos notveis e fatoraoes - 8h a c 2. Potenciaao, Radiciaao, Racionalizaao, Radicais Duplos, Funao Modular - 4h c c c c 3. Reviso de 1o grau: Proporoes, Regra de trs, bases de numeraao, d a c e c zimas peridicas, sistemas de equaoes, o c muitos e muitos problemas - 8h 4. Equaao do 2o grau e funao quadrtica - 10h c c a 5. Inequaoes(4h); Progresses (4h); Desigualdades entre as mdias (10h); Funoes e Polinmios (8h) - 26h c o e c o 6. Resoluao de Exerc c cios - aulas restantes Teoria dos N meros u 1. Divisores, nmeros primos, fatoraao em primos, regras de divisibilidade, MDC, Algoritmo de Euclides, MMC u c 10h 2. Princ pio da Induao - 6h c 3. Algoritmo da diviso e congruncias (6h); MDC e MMC (aprofundamento)(4h); Equaoes diofantinas (4h) - 14h a e c 4. Nmeros primos (6h); Funao parte inteira (4h) - 10h u c 5. Congruncias em Z (parte II) (6h); Ternos pitagricos; Equaoes Diofantinas II (6h) - 12h e o c 6. Teoermas de Fermat, Euler e Wilson - 6h 7. Teorema Chins dos Restos (4h); A funao phi de Euler (4h) - 8h e c 8. Resoluao de exerc c cios - aulas restantes Combinatria o 1. Princ pios da Contagem; Permutaoes e Permutaoes Circulares; Combinaoes; Combinaoes completas; Probac c c c bilidade - 16h aulas 2. Binmio de Newton, tringulo de Pascal e relaao de Stifel (opcional) o a c 3. O princ pio da casa e dos pombos - 6h 4. O princ pio da Incluso - Excluso (opcional) a a 5. Paridade; Jogos e Invariantes (10h); Teoria dos Grafos (8h); Seqncias recorrentes (8h) - 26h ue 6. Resoluao de Problemas - aulas restantes c

1o/2o anos
Carga horria proposta: 6h/aula por semana - curso de 40 semanas a Geometria Bibliograa:

55

Matemtica Elementar, vol 9; a Challenging Problems in Geometry, I.M. Yaglom Geometry Revisited, H.S.Coxeter Programa: 1. Paralelismo e Perpendicularismo. Congruncia de tringulos e desigualdade triangular - 8h e a 2. Semelhana de tringulos, teorema de Tales, teorema da bissetriz - 6h c a 3. Pontos Notveis no tringulo (I) (4h); Relaoes mtricas no tringulo retngulo. Teorema de Pitgoras. Relaoes a a c e a a a c mtricas no tringulo qualquer, lei dos senos, lei dos cossenos, relaao de Stewart, frmulas de rea (8h) - 12h e a c o a 4. Quadrilteros Inscrit a veis e o teorema de Ptolomeu; Teoremas de Ceva e Menelaus - 8h 5. Os c rculos do tringulo: inc a rculo, exinc rculos e circunc rculo (4h); Pontos Notveis (II): o tringulo rtico, o a a o tringulo medial, a reta de Euler e o c a rculo dos nove pontos, tringulo pedal, reta de Simpson, simtricos do a e ortocentro e outras propriedades; Teorema de Napoleo (10h) - 18h a 6. Geometria com Trigonometria - 8h 7. Transformaoes Geomtricas: Homotetia, Rotaao, Reexo, Translaao - 8h c e c a c 8. Geometria Anal tica para Olimp ada (opcional) Algebra 1. Produtos Notveis e Fatoraoes - 6h a c 2. Equaao do 2o grau e Funao quadrtica - 6h c c a 3. Inequaoes; Sequncias e Progresses - 6h c e o 4. Polinmios, relaoes de Girard (6h); Nmeros Complexos e ra o c u zes da unidade (6h) - 12h 5. Matrizes, determinantes e sistema lineares - 4h 6. Desigualdades (I) : Mdias, Cauchy-Schwarz (8h); Desigualdades (II): Rearranjo, Chebychev e outras elementares e (opcional)- 8h 7. Equaoes Funcionais - 12h c 8. Resoluao de problemas - aulas restantes c Teoria dos N meros u 1. Princ pio da Induao - 6h c 2. Algoritmo da Diviso e Congruncias (8h); MDC, MMC e Divisibilidade (6h) - 14h a e 3. Equaoes diofantinas (I) - 6h c 4. Nmeros primos; Funao parte inteira - 6h u c 5. Ternos Pitagricos; Teoremas de Euler-Fermat, Wilson e do Resto Chins; A funao phi de Euler - 12h o e c 6. Equaoes Diofantinas (II) - 8h c 7. Resoluao de Problemas- aulas restantes c Combinatria o 1. Contagem: princ pio fundamental, permutaoes, combinaoes, permutaoes circulares e combinaoes completas; c c c c Probabilidade - 12h

56

2. Princ pio da Incluso- Excluso; Nmeros binomiais, tringulo de Pascal e Binmio de Newton - 8h a a u a o 3. Princ pio da Casa e dos Pombos - 6h 4. Contagem dupla (8h); Sequncias recorrentes (8h) - 16h e 5. Teoria dos Grafos (I) (10h); Jogos e Invariantes (14h) - 24h 6. Aulas restantes: Exerc cios

You might also like